envi case digests 3.1

51
7/24/2019 Envi Case Digests 3.1 http://slidepdf.com/reader/full/envi-case-digests-31 1/51 1. G.R. No. 92605 July 16, 1991  APEX MINING CO., INC., vs. HONORABLE CANCIO C. GARCIA ASSISTANT EXECUTIVE SECRETARY, OFFICE OF THE PRESIDENT, HON. FULGENCIO S. FACTORAN, JR., SECRETARY OF ENVIRONMENT AND NATURAL RESOURCES, and MARCOPPER MINING CORPORATION, FACTS: The controversy in this case involves conflicting mining claims between herein petitioners Apex Mining Co., Inc., et al. and private respondent Marcopper Mining Corporation. The disputed area is inside a timberland area located at Moncayo, Davao del Norte and Cateel, Davao Oriental, consisting of 4,941.0 hectares. MARCOPPER was one of the first mining claimants in the disputed area, having registered its 16 claims through the filing of declarations of location pursuant to Presidential Decree No. 463, otherwise known as the Mineral Resources Development Decree. MARCOPPER, allegedly, after registering its mining claim, learned from the central office of the Bureau of Forest Development (BFD) that the disputed area is within an existing forest reservation known as the "Agusan-Davao-Surigao Forest Reserve" established by Proclamation No. 369 on February 27, 1931 by then Governor General Dwight F. Davis, and realizing the invalidity of its mining claims for having availed of a wrong procedure, abandoned its 16 mining claims and applied for a prospecting permit instead with the. The area covered by its application consisted of 4,941.0 hectares overlapping its abandoned mining claims. On July 1, 1985, the BFD issued to it a Permit to Prospect No. 755-123185. Upon verification from the records of the BMGS, Davao City Mineral District Office, it found that the area covered by its Permit to Explore No. 133 is also the subject of several claims/declarations of APEX. MARCOPPER filed with the BMGS a "Petition for Cancellation of Mining Claims and/or Small Scale Mining Permits" against APEX, alleging, among others and in substance, that the area covered by its Permit to Explore No. 133 and the declarations of locations/mining claims belonging to APEX are within an established and existing forest reservation and that the said mining claims/declarations of location of APEX are invalid for being violative of Presidential Decree No. 463 and its implementing rules and regulations since the acquisition of mining rights within a forest reserve is through the filing of application for a permit to prospect with the BFD and not through registration of declarations of location with the BMGS.  APEX filed a Motion to Dismiss Marcopper's petition, alleging, in substance, that their mining claims are not within any established or proclaimed forest reserve, and as such, the acquisition of mining lights thereto must be undertaken through the registration of declaration of location with the BMGS and not through the filing of an application for permit to prospect with the BFD; and that the permit to prospect and permit to explore issued to MARCOPPER are inoperative and of no legal force and effect. Motion to dismiss is granted. Accordingly, the Permit to Explore No. 133 of the MMC is hereby declared null and void. MARCOPPER appealed. DENR after due hearing, rendered the appealed decision the order was reversed and set aside. Accordingly Permit to Explore No. 133 of appellant Marcopper Mining Corporation is hereby declared valid and subsisting. The respective mining claims of appellees Apex Mining Corporation, et al., as well as Small Scale Mining Permit Nos. (X-1), 04 and (X-1), 05, are hereby declared null and void and/or inoperative. The Director, Bureau of

Upload: sarah-esther

Post on 20-Feb-2018

553 views

Category:

Documents


50 download

TRANSCRIPT

Page 1: Envi Case Digests 3.1

7/24/2019 Envi Case Digests 3.1

http://slidepdf.com/reader/full/envi-case-digests-31 1/51

1.

G.R. No. 92605 July 16, 1991 APEX MINING CO., INC., vs. HONORABLE CANCIO C. GARCIA ASSISTANT EXECUTIVESECRETARY, OFFICE OF THE PRESIDENT, HON. FULGENCIO S. FACTORAN, JR.,SECRETARY OF ENVIRONMENT AND NATURAL RESOURCES, and MARCOPPER MINING

CORPORATION,

FACTS:

The controversy in this case involves conflicting mining claims between herein petitioners ApexMining Co., Inc., et al. and private respondent Marcopper Mining Corporation. The disputedarea is inside a timberland area located at Moncayo, Davao del Norte and Cateel, DavaoOriental, consisting of 4,941.0 hectares.

MARCOPPER was one of the first mining claimants in the disputed area, having registered its16 claims through the filing of declarations of location pursuant to Presidential Decree No. 463,otherwise known as the Mineral Resources Development Decree. MARCOPPER, allegedly,

after registering its mining claim, learned from the central office of the Bureau of ForestDevelopment (BFD) that the disputed area is within an existing forest reservation known as the"Agusan-Davao-Surigao Forest Reserve" established by Proclamation No. 369 on February 27,1931 by then Governor General Dwight F. Davis, and realizing the invalidity of its mining claimsfor having availed of a wrong procedure, abandoned its 16 mining claims and applied for aprospecting permit instead with the. The area covered by its application consisted of 4,941.0hectares overlapping its abandoned mining claims. On July 1, 1985, the BFD issued to it aPermit to Prospect No. 755-123185.

Upon verification from the records of the BMGS, Davao City Mineral District Office, it found thatthe area covered by its Permit to Explore No. 133 is also the subject of severalclaims/declarations of APEX. MARCOPPER filed with the BMGS a "Petition for Cancellation of

Mining Claims and/or Small Scale Mining Permits" against APEX, alleging, among others and insubstance, that the area covered by its Permit to Explore No. 133 and the declarations oflocations/mining claims belonging to APEX are within an established and existing forestreservation and that the said mining claims/declarations of location of APEX are invalid for beingviolative of Presidential Decree No. 463 and its implementing rules and regulations since theacquisition of mining rights within a forest reserve is through the filing of application for a permitto prospect with the BFD and not through registration of declarations of location with the BMGS.

 APEX filed a Motion to Dismiss Marcopper's petition, alleging, in substance, that their miningclaims are not within any established or proclaimed forest reserve, and as such, the acquisitionof mining lights thereto must be undertaken through the registration of declaration of locationwith the BMGS and not through the filing of an application for permit to prospect with the BFD;and that the permit to prospect and permit to explore issued to MARCOPPER are inoperative

and of no legal force and effect. Motion to dismiss is granted. Accordingly, the Permit to ExploreNo. 133 of the MMC is hereby declared null and void.

MARCOPPER appealed. DENR after due hearing, rendered the appealed decision the orderwas reversed and set aside. Accordingly Permit to Explore No. 133 of appellant MarcopperMining Corporation is hereby declared valid and subsisting. The respective mining claims ofappellees Apex Mining Corporation, et al., as well as Small Scale Mining Permit Nos. (X-1), 04and (X-1), 05, are hereby declared null and void and/or inoperative. The Director, Bureau of

Page 2: Envi Case Digests 3.1

7/24/2019 Envi Case Digests 3.1

http://slidepdf.com/reader/full/envi-case-digests-31 2/51

Mines and Geo-Sciences, is hereby directed to cancel the registered mining claims and torevoke Small Scale Mining Permit.

The motion for reconsideration of said decision having been denied by the DENR, APEXappealed the case to the Office of the President, which through the Assistant ExecutiveSecretary for Legal Affairs, Cancio C. Garcia, rendered a decision, the instant appeal is hereby

dismissed for lack of merit. APEX filed a motion for reconsideration, but the same was denied.

Hence this petition.

Issues:

1.) Whether or not the disputed area is within an established and existing forest reservation?

2.) Whether or not disputed areas, being clearly within a forest reserve, open to mining location?

Ruling:

1.) the answer is in the affirmative. Proclamation No. 369 could not have, as a matter of law,established a forest reserve for the simple reason that it was issued not on the basis of ChapterXII, Title V of Act No. 2874 nor on the basis of Section 1826 of Act 2711 (the Revised

 Administrative Code which took effect on June 15, 1939), but on the basis of Section 8 of Act2874 which empowers the Governor-General only to reclassify lands of the public domain. Inconfirmation, Proclamation No. 369 does not even use the word "reserve" or "forest reserve‖.Section 8 of Act No. 2874, the former Public Land Act, the basis of Proclamation No. 369,provides —  Section 8. Only those lands shall be declared open to disposition or concessionwhich have been officially delimited and classified and, when practicable, surveyed, and whichhave not been reserved for public or quasi-public uses, nor appropriated by the Governor, nor inany manner become private property, not those on which a private right authorized andrecognized by this Act or any other valid law may be claimed, or which, having been reserved or

appropriated, have ceased to do so. However, the Governor-General may, for reasons of publicinterest, declare lands of the public domain open to disposition before the same have had theirboundaries established or been surveyed, or may, for the same reasons, suspend theirconcession or disposition by proclamation duly published or by Act of Legislature.

From the above-quoted provision, the Governor-General was specifically empowered not only todeclare lands of public domain open to disposition but also to suspend their concession ordisposition. Accordingly, withdrawal of a certain area to establish a forest reserve is, withoutquestion, within the power of the Governor-General. The then Governor-General Dwight F.Davis, in issuing Proclamation No. 369, withdrew from settlement or disposition the tracts ofland described therein to establish a forest reserve. The intention can be gleaned from the lastparagraph of Proclamation No. 369, which reads: From this reserve shall be considered

automatically excluded all areas which had already been certified and which in the future maybe proclaimed as classified and certified by the Director of Forestry as non-forest lands andapproved by the Secretary of Agriculture and Natural Resources.

2.) The disputed areas, being clearly within a forest reserve, are not open to mining location.Sections 8 and 13 of P.D. No. 463, as amended by P.D. No. 1385

Pursuant to P.D. No. 463, as amended, one can acquire mining rights within forest reserves byinitially applying for a permit to prospect with the Bureau of Forest and Development (BFD) and

Page 3: Envi Case Digests 3.1

7/24/2019 Envi Case Digests 3.1

http://slidepdf.com/reader/full/envi-case-digests-31 3/51

subsequently for a permit to explore with the Bureau of Mines and Geo-Sciences (BMGS). Suchprocedural requisites were complied with and undertaken by MARCOPPER after it hadascertained that its mining claims were found to be within the Agusan-Davao-Surigao ForestReserve. On the other hand, the mining claims and SSMPs of APEX being located within saidforest reserve are in violation of the law and therefore result in a failure to validly acquire miningrights.

Page 4: Envi Case Digests 3.1

7/24/2019 Envi Case Digests 3.1

http://slidepdf.com/reader/full/envi-case-digests-31 4/51

2.

G.R. Nos. 152613 & 152628APEX MINING CO., INC., vs. Southeast Mindanao Gold Mining Corp. (SEM) et. al

G.R. No. 152619-20

BALITE COMMUNAL PORTAL MINING COOPERATIVE vs. Southeast mindanao goldmining corp.

G.R. No. 152870-71THE MINES ADJUDICATION BOARD AND ITS MEMBERS, THE HON. VICTOR O. RAMOS(Chairman), UNDERSECRETARY VIRGILIO MARCELO (Member) and DIRECTORHORACIO RAMOS (Member) vs. southeast mindanao gold mining corporation

FACTS:

 A motion for reconsideration was filed by SEM. The Assailed Decision held that the assignmentof Exploration Permit (EP) 133 in favor of SEM violated one of the conditions stipulated in the

permit. It also ruled that the transfer of EP 133 violated Presidential Decree No. 463, whichrequires that the assignment of a mining right be made with the prior approval of the Secretaryof the Department of Environment and Natural Resources (DENR). Moreover, the AssailedDecision pointed out that EP 133 expired by non-renewal since it was not renewed before orafter its expiration. It likewise upheld the validity of Proclamation No. 297 absent any questionagainst its validity.

In view of this, and considering that under Section 5 of Republic Act No. 7942, otherwise knownas the ―Mining Act of 1995,‖ mining operations in mineral r eservations may be undertakendirectly by the State or through a contractor, the Court deemed the issue of ownership of priorityright over the contested Diwalwal Gold Rush Area as having been overtaken by the saidproclamation. Thus, it was held in the Assailed Decision that it is now within the prerogative of

the Executive Department to undertake directly the mining operations of the disputed area or toaward the operations to private entities including petitioners Apex and Balite, subject toapplicable laws, rules and regulations, and provided that these private entities are qualified.

 Apex, for its part, filed a Motion for Clarification of the Assailed Decision, praying that the Courtelucidate on the Decision‘s pronouncement that ―mining operations, are now, therefore withinthe full control of the State through the executive branch.‖ Moreover, Apex asks this Court toorder the Mines and Geosciences Board (MGB) to accept its application for an explorationpermit.

Balite echoes the same concern as that of Apex on the actual takeover by the State of themining industry in the disputed area to the exclusion of the private sector. In addition, Balite

prays for this Court to direct MGB to accept its application for an exploration permit.Camilo Banad, et al., likewise filed a motion for reconsideration and prayed that the disputedarea be awarded to them.

In the Resolution, the Court En Banc resolved to accept the instant cases.

Page 5: Envi Case Digests 3.1

7/24/2019 Envi Case Digests 3.1

http://slidepdf.com/reader/full/envi-case-digests-31 5/51

ISSUES:

1. Whether the transfer or assignment of Exploration Permit (EP) 133 by MMC to SEM wasvalidly made without violating any of the terms and conditions set forth in Presidential DecreeNo. 463 and EP 133 itself.

2. Whether Southeast Mindanao Mining Corp. acquired a vested right over the disputedarea, which constitutes a property right protected by the Constitution.

3. Whether the assailed Decision dated 23 June 2006 of the Third Division in this case iscontrary to and overturns the earlier Decision of this Court in Apex v. Garcia (G.R. No. 92605,16 July 1991, 199 SCRA 278).

4. Whether the issuance of Proclamation No. 297 declaring the disputed area as mineralreservation outweighs the claims of SEM, Apex Mining Co. Inc. and Balite Communal PortalMining Cooperative over the Diwalwal Gold Rush Area.

5. Whether the issue of the legality/constitutionality of Proclamation No. 297 was belatedly

raised.

HELD:

1. The assailed Decision did not overturn the 16 July 1991 Decision in Apex Mining Co.,Inc. v. Garcia. The former was decided on facts and issues that were not attendant in the latter,such as the expiration of EP 133, the violation of the condition embodied in EP 133 prohibitingits assignment, and the unauthorized and invalid assignment of EP 133 by MMC to SEM, sincethis assignment was effected without the approval of the Secretary of DENR;

2. SEM did not acquire vested right over the disputed area because its supposed right wasextinguished by the expiration of its exploration permit and by its violation of the condition

prohibiting the assignment of EP 133 by MMC to SEM. In addition, even assuming that SEMhas a valid exploration permit, such is a mere license that can be withdrawn by the State. Infact, the same has been withdrawn by the issuance of Proclamation No. 297, which places thedisputed area under the full control of the State through the Executive Department;

3. The approval requirement under Section 97 of Presidential Decree No. 463 applies tothe assignment of EP 133 by MMC to SEM, since the exploration permit is an interest in amining lease contract;4. The issue of the constitutionality and the legality of Proclamation No. 297 was raisedbelatedly, as SEM questions the same for the first time in its Motion for Reconsideration. Evenif the issue were to be entertained, the said proclamation is found to be in harmony with the

Constitution and other existing statutes;

5. The motion for reconsideration of Camilo Banad, et al. cannot be passed upon becausethey are not parties to the instant cases;

6. The prayers of Apex and Balite asking the Court to direct the MGB to accept theirapplications for exploration permits cannot be granted, since it is the Executive Department thathas the prerogative to accept such applications, if ever it decides to award the miningoperations in the disputed area to a private entity;

Page 6: Envi Case Digests 3.1

7/24/2019 Envi Case Digests 3.1

http://slidepdf.com/reader/full/envi-case-digests-31 6/51

4.

ATOK BIG-WEDGE MINING COMPANY, petitioner, vs. HON. INTERMEDIATE APPELLATECOURT and TUKTUKAN SAINGAN, respondents.

Facts:

Subject Land-41,296 square meters situated in the barrio of Lucnab, Itogon, Benguet.

Parties:

 A. ATOK BIG-WEDGE MINING COMPANY (claiming that the said parcel of land is a mineralland.)B. TUKTUKAN SAINGAN (claiming that the said parcel of land is agricultural.)

Contentions:

 Atok- they contended that the said parcel of land was being registered in the office of Mining

Recorder in 1921 and 1931 pursuant to Philippine Bill of 1902. It is about sixteen years beforeTUKTUKAN declared the land in question for taxation purposes and thirty four (34) years beforeprivate respondent filed the land registration proceedings in 1965. They also showed thepayment of annual assessment fees for the said land since 1931.

Tuktukan- who was 70 years old at the time he testified shows that he acquired the land fromhis father-in-law, Dongail, when he married his daughter; that he was then 18 years old; that atthe time of his acquisition, it was planted with camotes, casava, langka, gabi, coffee andavocados; that he lived on the land since his marriage up to the present; that he has beenpaying the taxes during the Japanese occupation and even before it; that he was neverdisturbed in his possession. Supporting his oral testimony, applicant [Tuktukan] submitted taxdeclarations x x x both dated March 20, 1948, the former for a rural land and the latter for urban

land and improvement therein.

Issue:

Whether or not the said parcel of land is a mineral land or an agricultural land.

History of mining Act1. Spanish Mining Law of 18672. Philippine Bill Of 1902 (American time) when the subject land had been registered3. Commonwealth Act No. 137 (under the 1935 Constitution)4. Executive Order 141 (Pres. Marcos 1968)5. President Decree No. 1214 (1977)

 All of the mining acts have a common provision which is the annual performance of labor orundertaking of improvements on the mine.

Held:

The SC ruled in favor of the Tuktukan in the reasons that:

Page 7: Envi Case Digests 3.1

7/24/2019 Envi Case Digests 3.1

http://slidepdf.com/reader/full/envi-case-digests-31 7/51

  A. Tuktukan have proven that he had in possession of the said land in a concept of an owner,continuously, open and uninterrupted for a period of more than 30 years.B. He had improve almost 90% of the said parcel of land.C. He had paid tax declaration of the said land since 1948 up to present.

It is evident that Atok had registered the land prior than Tuktukan but still the SC ruled in favorof Tuktukan for the reasons:

 A. Payment of annual assessment fee is not enough proof. There must be an annualperformance of labor or undertaking of improvements in the mine.B. When an ocular survey was made, it was evident that there was No improvements beingmade in the said land and there is any sign of mining had happened in the land. Hence, thepetition is DENIED.

Page 8: Envi Case Digests 3.1

7/24/2019 Envi Case Digests 3.1

http://slidepdf.com/reader/full/envi-case-digests-31 8/51

22.

G.R. No. 163101

BENGUET CORPORATION vs. DEPARTMENT OF ENVIRONMENT AND NATURALRESOURCES-MINES ADJUDICATION BOARD and J.G. REALTY AND MINING

CORPORATION

FACTS:

Benguet and J.G. Realty entered into a Royalty Agreement with Option to Purchase (RAWOP) ,wherein J.G. Realty was acknowledged as the owner of four mining claims with a total area of288.8656 hectares. The parties also executed a Supplemental Agreement.The mining claimswere covered by Mineral Production Sharing Agreement (MPSA) Application No. APSA-V-0009

 jointly filed by J.G. Realty as claim-owner and Benguet as operator.

 After some time, the Executive Vice-President of Benguet, Antonio N. Tachuling, issued a letterinforming J.G. Realty of its intention to develop the mining claims. However, J.G. Realty,

through its President, Johnny L. Tan, then sent a letter to the President of Benguet informing thelatter that it was terminating the RAWOP. The latter alleged that petitioner violated some of theprovisions of the RAWOP, specifically on non-payment of royalties and non-fulfillment ofobligations stipulated therein.

J.G. Realty filed a Petition for Declaration of Nullity/Cancellation of the RAWOP. POA issued aDecision, cancelling the RAWOP and its Supplemental Agreement. BENGUET wassubsequently excluded from the joint MPSA Application over the mineral claims. SubsequentMR was denied. Said decision was upheld by DENR-MAB.

Hence this instant petition.

ISSUE:

Whether or no the filing of the petition with the Supreme Court is proper.

HELD:

NO. the instant petition can be denied outright as Benguet resorted to an improperRemedy.

The last paragraph of Section 79 of Republic Act No. (RA) 7942 or the ―Philippine Mining Act of 1995‖ states, ―A petition for review by certiorari and question of law may be filed by theaggrieved party with the Supreme Court within thirty (30) days from receipt of the order or

decision of the [MAB].‖ 

The Revised Rules of Civil Procedure included Rule 43 to provide a uniform rule onappeals from quasi-judicial agencies. Under the rule, appeals from their judgments and finalorders are now required to be brought to the CA on a verified petition for review. A quasi-judicialagency or body has been defined as an organ of government, other than a court or legislature,which affects the rights of private parties through either adjudication or rule-making. MAB fallsunder this definition; hence, it is no different from the other quasi-judicial bodies enumeratedunder Rule 43.

Page 9: Envi Case Digests 3.1

7/24/2019 Envi Case Digests 3.1

http://slidepdf.com/reader/full/envi-case-digests-31 9/51

 The judicial policy of observing the hierarchy of courts dictates that direct resort fromadministrative agencies to this Court will not be entertained, unless the redress desired cannotbe obtained from the appropriate lower tribunals, or unless exceptional and compellingcircumstances justify availment of a remedy falling within and calling for the exercise of ourprimary jurisdiction, e.g., lack or abuse of discretion.

Thus Benguet should have filed the appeal with the CA first via Rule 43.

Petitioner having failed to properly appeal to the CA under Rule 43, the decision of the MAB hasbecome final and executory. On this ground alone, the instant petition must be denied.

Page 10: Envi Case Digests 3.1

7/24/2019 Envi Case Digests 3.1

http://slidepdf.com/reader/full/envi-case-digests-31 10/51

6.

Benguet Corp vs. Hon. Oscar Leviste in his capacity as Presiding Judge of RTC & HelenDizon-Reyes

FACTS:

- Helen Dizon Reyes executed SPA constituting Celestino Dizon, her father as Atty-In-Fact to transfer, assign & dispose other 11 mining claims.- AIF entered into agreement with Benguet Corp. and Dizon Mines whereby it was grantedthe right to explore, develop, exploit & operate the 57 mining claims including the 11 claims ofprivate respondent.- 3 mos. After the Deed of Ratification of Assignment with Dizon Mines was made, privaterespondent revoked the SPA & filed with the RTC for the declaration of Operations Agreementas null and void for physiological incapacity of Celestino |Dizon to give his consent thereto,which was granted by said court.- Benguet Corp. filed a Motion to Dismiss on the ff. grounds:1) The court is without jurisdiction over the subject matter and nature of the action.

2) The action is barred by prior judgment and laches3) The action to declare invalid the Deed of |Ratification has prescribed4) The venue of the action was improperly laid.

- The motion was denied by the trial court including its motion for reconsideration. Hence,this petition.

ISSUE:

WON the RTC has jurisdiction over cases involving cancellation and/or enforcement of miningcontracts?

HELD:

PD 1521 which took effect on January 16, 1978 vests the Bureau of Mines with jurisdictionalsupervision and control over all holders of mining claims or applicants for and/or grantees ofmining licenses, permits, leases and/or operators thereof, including mining service contractsand service contractors insofar as their mining activities are concerned. To effectively dischargeits task as the Government's arm in the administration and disposition of mineral resources,Section 7 of P.D. No. 1281 confers upon the Bureau quasi-judicial powers as follows:

"SEC. 7. In addition to its regulatory and adjudicative functions over companies, partnership orpersons engaged in mining exploration, development and exploitation, the Bureau of Minesshall have origins and exclusive jurisdiction to hear and decide cases involving:

xxx xxx xxx"(c) cancellation and/or enforcement of mining contracts due to the refusal of theclaimowner/operator to abide by the terms and conditions thereof.‖ 

In the case at bar, it is not disputed that the subject agreement is a mining contract and privaterespondent, in seeking a judicial declaration of its nullity, does not wish to abide by its terms andconditions. These elements alone bring the action within the ambit of Section 7 of P.D. 1281.Whatever the basis for the refusal to abide by the contract's terms and conditions, the basic

Page 11: Envi Case Digests 3.1

7/24/2019 Envi Case Digests 3.1

http://slidepdf.com/reader/full/envi-case-digests-31 11/51

issue remains one of its cancellation, which is precisely what P.D. No. 1281 places within theexclusive original jurisdiction of the Bureau.

The reason underlying such refusal is indeed an irrelevant matter insofar as jurisdictionalcompetence is concerned, for to make jurisdiction dependent thereon would not only be"ratifying two judicial bodies exercising jurisdiction over an essentially the same subject matter a

situation analogous to split jurisdiction which is obnoxious to the orderly administration of justice" but also clearly ignoring the object of P.D. 1281 to make the adjudication of miningcases a purely administrative matter.

In the light of our ruling that the jurisdiction over private respondent's action to annul theOperations Agreement pertaining to the Bureau of Mines and Geo-Sciences rather than theregional trial court, the question of venue becomes immaterial.

Considering further that the other issues raised by petitioner, namely res judicata, laches andprescription are factual matters which are not only improper in a petition for certiorari but which,more importantly, petitioner failed to substantiate, no ruling on these issues need be made.

WHEREFORE, the instant petition is GRANTED. The assailed orders of March 26, 1982 andJune 20, 1983 are set aside and Civil Case No. Q-30171 of the Regional Trial Court of QuezonCity, Branch XCVII, is ordered DISMISSED. This decision is immediately executory. Costsagainst private respondent

Page 12: Envi Case Digests 3.1

7/24/2019 Envi Case Digests 3.1

http://slidepdf.com/reader/full/envi-case-digests-31 12/51

7.

G.R. No. L-24757 October 25, 1967MARCOS B. COMILANG vs. HON. GENEROSO A. BUENDIA

FACTS:

 About the year 1908, Nicolas Comilang staked a mining claim known as the "Bua FractionMineral Claim" over a parcel of land in Tuding, Benguet, Mountain Province, with an area of76,809 square meters, more or less. His exploration works in the mining claim did not last forlong, for he abandoned it, and stopped the exploration, but he continued to live in the house hebuilt on a portion of the land with his wife, brothers and sisters.In the year 1918, Macario Comilang also settled on a portion of the land with an area of aboutone (1) hectare, for residential and agricultural purposes. After his death, his daughter, FabianaComilang Perez remained to live in the house built by her father on the land. Still later, otherrelatives of the old Nicolas Comilang settled and built their own houses over other portions ofthe land, one of which houses was acquired by Abdon Delenela who now resides on the landwith the other Comilang heirs.Surface rights over the area embraced in the original Bua Fraction Mineral Claim of Nicolas

Comilang soon became the subject of litigation in the Court of First Instance of Baguio City (CivilCase No. 250 — Action to Quiet Title), instituted by the heirs of Guillerma, Marcelina, Julian,Timoteo, Melecio and Macario, all surnamed Comilang, against appellant herein MarcosComilang who claimed to have bought the rights and interest of Nicolas Comilang in the oldmining claim. In a decision rendered in said case No. 250, dated November 26, 1952, the courtdismissed both claims of ownership of the plaintiffs and the defendant and declared the areapublic land. The court, however, recognized the possession of the parties over certain specifiedportions of the area, among which was an area of about one and one-half (1-1/2) hectares inpossession of Marcos Comilang, which has been declared for taxation purposes in his name.This decision was affirmed by the Court of Appeals in CA-G.R. No. 11157-R on October 29,1955.In the same year, the 1-1/2 hectares of land occupied by Marcos Comilang, then declared under

Tax Declaration No. 4771 in his name, was levied upon and sold at public auction by the sheriffof Mountain Province to satisfy a judgment for a sum of money obtained by the spouses JoseColoma and Eugenia Rumbaoa against Marcos Comilang in the Court of First Instance ofBaguio, in Civil Case No. 1433. The judgment creditors were the purchasers at the auction sale,and a certificate of sale was executed in their favor by the sheriff on June 1, 1957.In the meantime, an application for lode patent covering the Bua Fraction Mineral Claim wasfiled with the Bureau of Mines. Abdon Delenela and his co-heirs filed their opposition to theapplication. Pending the controversy before the Bureau of Mines, Delenela and his co-heirsinstituted an action for determination of their rights on the land in the Court of First Instance ofBaguio City, docketed as Civil Case No. 735. The parties submitted an amicable settlementrecognizing co-ownership among themselves of the Bua Mineral Claim. In a decision renderedin said Case No. 735, dated March 3, 1958, the court awarded one-half in undivided share in

the mineral claim in favor of Marcos Comilang, and the other half also in undivided share infavor of Abdon Delenela and co-heirs.Later, in the exercise of their right as co-owners, Abdon Delenela and Guillermo Perez, with theknowledge and conformity of Marcos Comilang, redeemed and bought from the Colomaspouses, the latter's rights, title, interest and claim to the 1-1/2 hectares of land acquired underthe certificate of sale thereof executed in the latter's favor by the sheriff on June 1, 1957. Thisredemption sale took place on June 11, 1958.On February 9, 1959, the Director of Mines recommended the issuance of a lode patent overthe Bua Mineral Claim in favor of Marcos Comilang, Delenela, and the other claimants in the

Page 13: Envi Case Digests 3.1

7/24/2019 Envi Case Digests 3.1

http://slidepdf.com/reader/full/envi-case-digests-31 13/51

proportion of one-half (1/2) in undivided share in favor of Marcos Comilang, and the other one-half (1/2) also in undivided share in favor of Delenela and the other heirs pursuant to thedecision of March 3, 1958, aforementioned, in Civil Case No. 735.On August 12, 1959, upon motion of Abdon Delenela and Perez, who have thus acquired andsucceeded to the rights of the Coloma spouses on the 1-1/2 hectares, the Municipal Court ofBaguio City issued a writ of possession in their favor directing the sheriff of Mountain Province

to evict Marcos Comilang and his wife from the 1-1/2 hectares of land sold in the execution sale.In a petition for certiorari with preliminary injunction filed in the Court of First Instance of BaguioCity, docketed as Civil Case No. 897, Maxima Nieto de Comilang, wife of Marcos Comilang,questioned the power of the Municipal Court to issue said writ of possession on two grounds,namely: (1) that conjugal property had been levied upon and sold in the execution sale, and hershare therein is affected; and (2) that there can be no severance of surface rights over a mineralclaim located under the Philippine Bill of 1902, and petitioner argued that the sheriff could nothave validly sold the surface rights in the execution sale of June 1, 1957. On February 23, 1961,the court rendered a decision in said case, holding that the writ of possession issued by therespondent Municipal Judge was within his competence and jurisdiction. On appeal to theSupreme Court, docketed as G. R. No. L-18897, a decision was rendered on March 31, 1964,the dispositive portion of which is as follows:

For the foregoing considerations the judgment appealed from is hereby affirmed insofar as itdenies the petition of Maxima Nieto de Comilang to exclude from the sale, or annul the sale onexecution of the residential lot formerly owned by her husband, of 1-1/2 hectares covered in thefinal certificate of sale; but that part of the appealed decision holding that the sale at publicauction included the 1/2 undivided share of Marcos Comilang to the Bua Mineral Claim, ishereby set aside and said mineral rights of Marcos Comilang are hereby declared free from theexecution or sale on execution.The decision having become final, Abdon Delenela and Guillermo Perez reiterated their motionin the Municipal Court of Baguio City in Civil Case No. 1433, praying that an alias writ ofpossession be issued to evict Marcos Comilang and his wife from the 1-1/2 hectares of land inquestion. On August 11, 1964, over the objection of Marcos Comilang, the court issued the writprayed for.

For a second time, a petition for certiorari and mandamus with preliminary injunction wasinstituted by Marcos Comilang in the Court of First Instance of Baguio City seeking theannulment of the order granting the alias writ of possession in favor of Delenela and Perez, andagain the Court of First Instance of Baguio threw out the petition in its order dated October 22,1964. The court expressed its views in the following rationale:The one and one-half hectares of land referred to therein (S.C. decision) is the same parcel ofland and house above-described which was already sold at public auction to the respondents,Guillermo Perez and Abdon Delenela.The said judgment is res adjudicata and the consequent execution, and the writ of possession isbut its necessary consequence.

 All the authorities cited by the petitioner were no longer of any value because they werenecessarily passed upon and disposed of in the course of finally deciding the case.

Wherefore, the petition for certiorari is hereby denied.Marcos Comilang is now before Us on appeal from this last decision.

 Appellant contends that the lower court erred in denying his petition on the ground of resadjudicata, arguing that it was his wife Maxima Nieto de Comilang, and not be, the party in theformer case appealed to the Supreme Court in G. R. No. L-18897. Therefore, it is claimed, oneof the requisites of res adjudicata is lacking. We find no merit in the argument. As husband andwife and before the dissolution of their marital union, their interest in the said property is oneand the same. The fact that the wife was the party in the former case while it is the husbandwho is the petitioner in the instant case, when admittedly both actions were instituted for the

Page 14: Envi Case Digests 3.1

7/24/2019 Envi Case Digests 3.1

http://slidepdf.com/reader/full/envi-case-digests-31 14/51

protection of their common interest therein, is no argument to the proposition that there is noidentity of parties in these cases. Such identity of interest is enough to hold that they are privy toone another, having a common interest in the property. Neither is it tenable to contend that theissue involved in the two cases are not identical. It cannot be disputed that in both cases, themain relief sought is the annulment of writs of possession issued by the Municipal Court ofBaguio City directing the sheriff concerned to evict the spouses Comilang from the land, and the

questions involved in both cases pertain to the legality or validity of those writs aforementioned.In the decision in L-18897, this Court sustained the validity of the execution sale. There can beno doubt, therefore, that the judgment in the former case is binding in the instant proceeding.It is argued further by the appellant that the final certificate of sale conveying the land describedin Tax Declaration No. 4771 to the purchasers in the execution sale is not a valid disposition ofa portion of the public domain, and specially in view of the subsequent issuance of a minerallode patent over the Bua Mineral Claim by the Director of Mines (Patent issued on November 7,1966) whereby full ownership not only of the minerals therein but also of the surface groundhave been conveyed to the patentee thereof, and, therefore, the Municipal Court of Baguio Citymay no longer eject them from the land.We do not agree with the contention of the appellant.The Court has not overlooked the doctrines heavily relied upon by the appellant that the

moment the locator discovered a valuable mineral deposit on the land located, and perfected hislocation in accordance with the provisions of the Philippine Bill of 1902, the power of theGovernment to deprive him of the exclusive right to possession of the located claim was gone,the land had become mineral land and they were excepted from the lands that could be grantedto any other person (McDaniel v. Apacible and Cuisia, 42 Phil. 749, 756); and that when alocation of a mining claim is perfected under said law, it has the effect of a grant by the UnitedStates of the right of present possession, with the right to the exclusive enjoyment of all thesurface ground as well as of all the minerals within the lines of the claim (Gold Creek MiningCorporation v. Rodriguez, 66 Phil. 259). We are also cognizant of the rule invoked by theappellant that when circumstances have arisen subsequent to the remanding of the record fromthe Supreme Court to the trial court, a stay of execution may be allowed on grounds which arein their nature peculiarly equitable, as for instance, to give defendant an opportunity to set off a

claim against plaintiff (Chua A. H. Lee v. Mapa, 51 Phil. 624); or when after judgment has beenrendered and it has become final, facts and circumstances transpire which rendered itsexecution impossible and unjust, the interested party may ask the court to alter or modify the

 judgment to harmonize the same with justice and the facts (De la Costa v. Cleopas, 67 Phil.686; Realiza v. Duarte, L-25027, L-20528 & L-20529, August 31, 1967); and this remains true,notwithstanding affirmance of the judgment by the Supreme Court, which imparts no higherquality than to a final judgment unappealed from, except that it cannot be questioned orreviewed (Chua A. H. Lee v. Mapa, supra). However, these authorities, by no means, render theargument of herein appellant unassailable . There are factual differences in the settings of thecase cited and the one at bar, the equities of which require the application of a different rule.To begin with, the 1-½ hectares portions of the Bua Fraction Mineral Claim described in TaxDeclaration No. 4771 in the name of herein appellant was levied upon and sold at public auction

to satisfy the money judgment against him in Civil Case No. 1433 of the Municipal Court ofBaguio City, and the corresponding certificate of sale was issued in favor of the judgmentcreditors. Interest acquired under like certificates of sale alone has been described as morethan a lien on the property, more than an equitable estate, an inchoate legal title to the property.(21 Am. Jur., section 264, p. 133). The validity of that sale was questioned when the MunicipalCourt ordered the eviction of appellant from the land sold on execution, and the Supreme Courtdeclared in L-18897 that the sale was valid. The sale operated to divest appellant of his rights tothe land which vested in the purchasers at the auction sale. The parties herein subsequentlylitigated their rights to the mineral claim in Civil Case No. 735 of the Court of First Instance of

Page 15: Envi Case Digests 3.1

7/24/2019 Envi Case Digests 3.1

http://slidepdf.com/reader/full/envi-case-digests-31 15/51

Baguio City, and on the basis of their amicable agreement (appellant was a party in the case),the court declared the Bua Mineral Claim co-ownership property of the parties thereto "exceptthe improvements existing thereon" (p. 9, appellant's petition). There is no room for doubt,therefore, that the right to possess or own the surface ground is separate and distinct from themineral rights over the same land. And when the application for lode patent to the mineral claimwas prosecuted in the Bureau of Mines, the said application could not have legally included the

surface ground sold to another in the execution sale. Consequently, We have to declare that thepatent procured thereunder, at least with respect to the 1-½ hectares sold in execution pertainsonly to the mineral right and does not include the surface ground of the land in question.Viewed from another perspective, We have arrived at the same conclusion. In his letter to theSecretary of Agriculture and Natural Resources, dated February 9, 1959, recommending theapproval of Mineral Lode Patent No. V-24, the Director of Mines said that applicants MarcosComilang, et al., had acquired vested rights on the Bua Fraction Mineral Claim before theConstitution of the Philippines was approved on November 15, 1935. Under the doctrines laiddown in McDaniel v. Apacible, and in Gold Creek Mining v. Rodriguez, supra, said vested rightsinclude the ownership of both the minerals and the surface ground; that such was the locator'sright before as well as after the issuance of the patent; and that such was vested propertyalthough fee remains in the Government until patent issues. Such vested right of herein

appellant passed to the appellees under the sale on execution aforementioned of the 1-½hectares portion of the mineral claim. The subsequent issuance of the Lode Patent to the entirearea of the Bua Mineral Claim did not militate against that acquired rights, for Sec. 45 of thePhilippine Bill of 1902 expressly provides that nothing in said Act shall be deemed to impair anylien which may have attached in any way whatever prior to the issuance of the patent.Moreover, it is significant to note that the very Lode Patent No. V-24 aforementioned expresslydeclares on its face that "the mining premises hereby conveyed shall be held subject to allvested lights and accrued rights", the legal import of which is that the patentee MarcosComilang, shall hold the1½ hectares portion of the area embraced in the patent as described inthe Tax Declaration No. 4771, in trust for the appellees.

 Apart and independent of the statute, there is a rule in American Law known as the "Doctrine ofRelation", to the effect "that all parts and ceremonies necessary to complete a conveyance shall

be taken together as one act, and operate from the substantial part by relation." This"substantial part" is recognized as the "original act" which is to be preferred, and to this allsubsequent acts are to have relation. This doctrine of relation appears to have been oftenapplied to the adjudication of real actions by American courts.The case of Landes v. Brant, 10 How. 348, U. S. 13 Law ed., 449, broadly asserts this doctrineof relation. In that case, a Spanish claim of land was acquired by Clamorgan under Dodier, theoriginal claimant, by virtue of ten consecutive years possession prior to December 20, 1903.Such claim was authorized by the Act of Congress. Clamorgan was entitled to a patent by virtueof a certificate of confirmation made by commissioners. His petition for such confirmation wasfiled in December, 1805. In 1808 judgment was recovered against Clamorgan, the claim wassold and the sheriff's deed executed to McNair. It was held that the execution sale passed to thepurchaser all the title that could have passed from Clamorgan to McNair by a quitclaim deed;

that applying thedoctrine of relation and taking all the parts and ceremonies necessary tocomplete the title together as one act, then the confirmation of 1811 and the patent of 1845must be taken to relate to the first act; that of filing the claim in 1805. On this assumption,intermediate conveyances made by the confirmed or by the sheriff on his behalf, of a date afterthe first substantial act, are covered by the legal title and pass that title to the alienee. And onthis ground, the deed made by the sheriff to McNair is valid. This doctrine has been applied in agreat number of decisions.

 Applying the same rule to the case before Us, it is seen that the original act that ripened intoMineral Lode Patent No. V-24 was the location of the mineral claim and the recording thereof in

Page 16: Envi Case Digests 3.1

7/24/2019 Envi Case Digests 3.1

http://slidepdf.com/reader/full/envi-case-digests-31 16/51

the Mining Recorder of Mt. Province sometime in 1922. Vested right to the property accrued tothe locator before 1935, although patent was issued only recently (November 7, 1966). ThisPatent cannot nullify the intermediate conveyance of that right in the execution sale of 1958 toherein appellees.Finally, the argument that the proceedings for the issuance of a writ of possession, as has beenresorted to by the appellees, is not the proper court procedure, the appellant intimating that it

should be by a proper action. The contention does not deserve serious consideration. Thecorresponding rights of the parties to the property in question had been ventilated in the variouscases affecting it, and the decisions in those cases have sustained the validity of the sale. It isnow a matter of right on the part of the appellees to be placed in possession of the land by clearmandate of Sec. 35, Rule 39 of the Rules of Court which requires that upon execution anddelivery of the final deed of sale in execution the possession of the property shall be given to thepurchaser or last redemptioner unless a third party is actually holding the property adversely tothe judgment debtor. As this Court said in Tan Soo Huat v. Ongwico, 63 Phil. 747:There is no law in this jurisdiction whereby the purchaser at a sheriff's sale of real property isobliged to bring a separate and independent suit for possession after the one-year period forredemption has expired and after he has obtained the sheriff's final certificate of sale. There isneither legal ground nor reason of public policy precluding the court from ordering the sheriff in

this case to yield possession of the property purchased at public auction where it appears thatthe judgment debtor is the one in possession thereof and no rights of third persons are involved.WHEREFORE, the decision appealed from is affirmed. Costs against appellants.

Page 17: Envi Case Digests 3.1

7/24/2019 Envi Case Digests 3.1

http://slidepdf.com/reader/full/envi-case-digests-31 17/51

8.

G.R. No. 157882 March 30, 2006

Didipio Earth-Savers’ Multi-Purpose Association, Inc. (DESAMA) Et al. vs.Elisea gozun, et al.

FACTS:

 A petition for mandamus and prohibition assailing the constitutionality of the Philippine Mining Act of 1995, together with the IRR issued by the DENR Administrative Order No. 96-40, s.1996(DAO 96-40) and of the Financial and Technical Assistance Agreement (FTAA) enteredinto on20 June 1994 by the Republic of the Philippines and Arimco Mining Corporation (AMC), acorporation established under the laws of Australia and owned by its nationals. After severalunsuccessful actions to cancel the FTAA agreement with the government, the petitioners finallysubmitted a petition to the court. In their memorandum petitioners pose whether or not Republic

 Act No. 7942 and the CAMC FTAA are void because they allow the unjust and unlawful takingof property without payment of just compensation , in violation of Section 9, Article III of the

Constitution issues, among others issues.

ISSUE:

Whether there has been an actual controversy or issue with respect to the unlawful and unjusttaking of property without payment of just compensation.

HELD:

Public respondents are of the view that petitioners eminent domain claim is not ripe foradjudication as they fail to allege that CAMC has actually taken their properties nor do theyallege that their property rights have been endangered or are in danger on account of CAMC‘s

FTAA. In effect, public respondents insist that the issue of eminent domain is not a justiciablecontroversy which this Court can take cognizance of. A question is considered ripe foradjudication when the act being challenged has had a direct adverse effect on the individualchallenging it. However, the court cannot await the adverse consequences of the law in order toconsider the controversy actual and ripe for judicial intervention.

 Actual eviction of the land owners and occupants need not happen for this Court to intervene.By the mere enactment of the questioned law or the approval of the challenged act, the disputeis said to have ripened into a judicial controversy even without any other overt act. Indeed, evena singular violation of the Constitution and/or the law is enough to awaken judicial duty.Nevertheless, the petition was still dismissed due to the baseless contention of the issuessubmitted. The FTAA was in full compliance with the necessary requirements of the law and

Constitution. The allegation of the lack of payment of just compensation was dismissed sincethe court has had authority in eminent domain cases to make sure the proper amount wasestablished regardless of the fact that there would be an intervention from an executivedepartment or legislature to make any initial determination of the amount.

Page 18: Envi Case Digests 3.1

7/24/2019 Envi Case Digests 3.1

http://slidepdf.com/reader/full/envi-case-digests-31 18/51

9.

JORGE GONZALES and PANEL OF ARBITRATORS vs. CLIMAX MINING LTD., CLIMAX-ARIMCO MINING CORP. and AUSTRALASIAN PHILIPPINES MINING INC., G.R. No.161957, January 22, 2007

Facts:

This is a consolidation of two petitions rooted in the same disputed Addendum Contract enteredinto by the parties.

In one case, the Court held that the DENR Panel of Arbitrators had no jurisdiction over thecomplaint for the annulment of the Addendum Contract on grounds of fraud and violation of theConstitution and that the action should have been brought before the regular courts as itinvolved judicial issues.

Gonzales averred that the DENR Panel of Arbitrators Has jurisdiction because the case involvesa mining dispute that properly falls within the ambit of the Panel‘s authority.  

Respondents Climax Mining Ltd., et al., on the other hand, seek reconsideration/clarification onthe decision holding that the case should not be brought for arbitration under R.A. No. 876.They argued that the arbitration clause in the Addendum Contract should be treated as anagreement independent of the other terms of the contract, and that a claimed rescission of themain contract does not avoid the duty to arbitrate.

On another case, Gonzales challenged the order of the RTC requiring him to proceed with thearbitration proceedings while the complaint for the nullification of the Addendum Contract waspending before the DENR Panel of Arbitrators. He contended that any issue as to the nullity,inoperativeness, or incapability of performance of the arbitration clause/agreement raised byone of the parties to the alleged arbitration agreement must be determined by the court prior to

referring them to arbitration.

While Climax-Arimco contended that an application to compel arbitration under Sec. 6 of R.A.No. 876 confers on the trial court only a limited and special jurisdiction, i.e., a jurisdiction solelyto determine (a) whether or not the parties have a written contract to arbitrate, and (b) if thedefendant has failed to comply with that contract.

Issue: 

Whether or not arbitration is proper even though issues of validity and nullity of the AddendumContract and, consequently, of the arbitration clause were raised.

Ruling: 

Positive.

In La Naval Drug Corporation v. Court of Appeals, the Court held that R.A. No. 876 explicitlyconfines the court's authority only to the determination of whether or not there is an agreementin writing providing for arbitration. In the affirmative, the statute ordains that the court shall issuean order "summarily directing the parties to proceed with the arbitration in accordance with theterms thereof." If the court, upon the other hand, finds that no such agreement exists, "the

Page 19: Envi Case Digests 3.1

7/24/2019 Envi Case Digests 3.1

http://slidepdf.com/reader/full/envi-case-digests-31 19/51

proceeding shall be dismissed." The cited case also stressed that the proceedings are summaryin nature.

Implicit in the summary nature of the judicial proceedings is the separable or independentcharacter of the arbitration clause or agreement.

The doctrine of separability or severability enunciates that an arbitration agreement isindependent of the main contract. The arbitration agreement is to be treated as a separateagreement and the arbitration agreement does not automatically terminate when the contract ofwhich it is part comes to an end.

The separability of the arbitration agreement is especially significant to the determination ofwhether the invalidity of the main contract also nullifies the arbitration clause. Indeed, thedoctrine denotes that the invalidity of the main contract, also referred to as the ―container‖ contract, does not affect the validity of the arbitration agreement. Irrespective of the fact that themain contract is invalid, the arbitration clause/agreement still remains valid and enforceable.

The validity of the contract containing the agreement to submit to arbitration does not affect the

applicability of the arbitration clause itself. A contrary ruling would suggest that a party‘s mererepudiation of the main contract is sufficient to avoid arbitration. That is exactly the situationthat the separability doctrine, as well as jurisprudence applying it, seeks to avoid.

The Court added that when it declared that the case should not be brought for arbitration, itshould be clarified that the case referred to is the case actually filed by Gonzales before theDENR Panel of Arbitrators, which was for the nullification of the main contract on the ground offraud, as it had already been determined that the case should have been brought before theregular courts involving as it did judicial issues.

Page 20: Envi Case Digests 3.1

7/24/2019 Envi Case Digests 3.1

http://slidepdf.com/reader/full/envi-case-digests-31 20/51

10.

G.R. No 127882 January 27, 2004La Bugal-B'Laan Tribal Assn vs. Ramos

FACTS:

On July 25, 1987, then President Corazon C. Aquino issued Executive Order (E.O.) No.2796 authorizing the DENR Secretary to accept, consider and evaluate proposals from foreign-owned corporations or foreign investors for contracts or agreements involving either technical orfinancial assistance for large-scale exploration, development, and utilization of minerals, which,upon appropriate recommendation of the Secretary, the President may execute with the foreignproponent.

On March 3, 1995, then President Fidel V. Ramos approved R.A. No. 7942 to "governthe exploration, development, utilization and processing of all mineral resources." R.A. No. 7942defines the modes of mineral agreements for mining operations, outlines the procedure for theirfiling and approval, assignment/transfer and withdrawal, and fixes their terms. Similar provisions

govern financial or technical assistance agreements.

On April 9, 1995, 30 days following its publication on March 10, 1995 in Malaya andManila Times, two newspapers of general circulation, R.A. No. 7942 took effect. Shortly beforethe effectivity of R.A. No. 7942, however, or on March 30, 1995, the President entered into anFTAA with WMCP covering 99,387 hectares of land in South Cotabato, Sultan Kudarat, Davaodel Sur and North Cotabato.

On August 15, 1995, then DENR Secretary Victor O. Ramos issued DENR Administrative Order (DAO) No. 95-23, s. 1995, otherwise known as the Implementing Rulesand Regulations of R.A. No. 7942. This was later repealed by DAO No. 96-40, s. 1996 whichwas adopted on December 20, 1996.

On January 10, 1997, counsels for petitioners sent a letter to the DENR Secretarydemanding that the DENR stop the implementation of R.A. No. 7942 and DAO No. 96-40, givingthe DENR fifteen days from receipt to act thereon. The DENR, however, has yet to respond oract on petitioners' letter.Petitioners claim that the DENR Secretary acted without or in excess of jurisdiction.They pray that the Court issue an order:

(a) Permanently enjoining respondents from acting on any application for Financial or Technical Assistance Agreements;

(b) Declaring the Philippine Mining Act of 1995 or Republic Act No. 7942 as unconstitutional and

null and void;

(c) Declaring the Implementing Rules and Regulations of the Philippine Mining Act contained inDENR Administrative Order No. 96-40 and all other similar administrative issuances asunconstitutional and null and void; and

(d) Cancelling the Financial and Technical Assistance Agreement issued to Western MiningPhilippines, Inc. as unconstitutional, illegal and null and void.

Page 21: Envi Case Digests 3.1

7/24/2019 Envi Case Digests 3.1

http://slidepdf.com/reader/full/envi-case-digests-31 21/51

ISSUE:

Whether or not Republic Act No. 7942 is unconstitutional.

HELD:

The Court finds the following provisions of R.A. No. 7942 to be violative of Section 2, Article XIIof the Constitution and hereby declares unconstitutional and void:

(1) The proviso in Section 3 (aq), which defines "qualified person," to wit:Provided, That a legally organized foreign-owned corporation shall be deemed a qualifiedperson for purposes of granting an exploration permit, financial or technical assistanceagreement or mineral processing permit.

(2) Section 23, which specifies the rights and obligations of an exploration permittee, insofar assaid section applies to a financial or technical assistance agreement,

(3) Section 33, which prescribes the eligibility of a contractor in a financial or technical

assistance agreement;

(4) Section 35, which enumerates the terms and conditions for every financial or technicalassistance agreement;

(5) Section 39, which allows the contractor in a financial and technical assistance agreement toconvert the same into a mineral production-sharing agreement;

(6) Section 56, which authorizes the issuance of a mineral processing permit to a contractor in afinancial and technical assistance agreement;

The following provisions of the same Act are likewise void as they are dependent on the

foregoing provisions and cannot stand on their own:

(1) Section 3 (g), which defines the term "contractor," insofar as it applies to a financial ortechnical assistance agreement.

Section 34, which prescribes the maximum contract area in a financial or technical assistanceagreements;

Section 36, which allows negotiations for financial or technical assistance agreements;

Section 37, which prescribes the procedure for filing and evaluation of financial or technical

assistance agreement proposals;

Section 38, which limits the term of financial or technical assistance agreements;

Section 40, which allows the assignment or transfer of financial or technical assistanceagreements;

Section 41, which allows the withdrawal of the contractor in an FTAA;

Page 22: Envi Case Digests 3.1

7/24/2019 Envi Case Digests 3.1

http://slidepdf.com/reader/full/envi-case-digests-31 22/51

The second and third paragraphs of Section 81, which provide for the Government's share in afinancial and technical assistance agreement; and

Section 90, which provides for incentives to contractors in FTAAs insofar as it applies to saidcontractors;

When the parts of the statute are so mutually dependent and connected as conditions,considerations, inducements, or compensations for each other, as to warrant a belief that thelegislature intended them as a whole, and that if all could not be carried into effect, thelegislature would not pass the residue independently, then, if some parts are unconstitutional, allthe provisions which are thus dependent, conditional, or connected, must fall with them.

Page 23: Envi Case Digests 3.1

7/24/2019 Envi Case Digests 3.1

http://slidepdf.com/reader/full/envi-case-digests-31 23/51

11.

LEPANTO CONSOLIDATED MINING CO.,vs.WMC RESOURCES INT’L. PTY. LTD., WMC PHILIPPINES, INC. and SAGITTARIUS MINES,

INC.,

FACTS:

Philippine Government and WMC Philippines, the local wholly-owned subsidiary of WMCResources International Pty. Ltd. (WMC Resources) executed a Financial and Technical

 Assistance Agreement, denominated as the Columbio FTAA No. 02-95-XI (Columbio FTAA) forthe purpose of large scale exploration, development, and commercial exploration of possiblemineral resources in an initial contract area of 99,387 hectares located in the provinces of SouthCotabato, Sultan Kudarat, Davao del Sur, and North Cotabato in accordance with ExecutiveOrder No. 279 and Department Administrative Order No. 63, Series of 1991.

The Columbio FTAA is covered in part by 156 mining claims held under various MineralProduction Sharing Agreements (MPSA) by Southcot Mining Corporation, Tampakan Mining

Corporation, and Sagittarius Mines, Inc. (collectively called the Tampakan Companies), inaccordance with the Tampakan Option Agreement entered into by WMC Philippines and theTampakan Companies on 25 April 1991, as amended by Amendatory Agreement dated 15 July1994, for purposes of exploration of the mining claims in Tampakan, South Cotabato. TheOption Agreement, among other things, provides for the grant of the right of first refusal to theTampakan Companies in case WMC Philippines desires to dispose of its rights and interests inthe mining claims covering the area subject of the agreement.

WMC Resources subsequently divested itself of its rights and interests in the Columbio FTAA,and on 12 July 2000 executed a Sale and Purchase Agreement with petitioner Lepanto over itsentire shareholdings in WMC Philippines, subject to the exercise of the Tampakan Companies‘exercise of their right of first refusal to purchase the subject shares. On 28 August 2000,

petitioner sought the approval of the 12 July 2000 Agreement from the DENR Secretary.In the interim, on 10 January 2001, contending that the 12 July Agreement between petitionerand WMC Philippines had expired due to failure to meet the necessary preconditions for itsvalidity, WMC Resources and the Tampakan Companies executed another Sale and Purchase

 Agreement, where Sagittarius Mines, Inc. was designated assignee and corporate vehicle whichwould acquire the shareholdings and undertake the Columbio FTAA activities. On 15 January2001, Sagittarius Mines, Inc. increased its authorized capitalization to P250 million.Subsequently, WMC Resources and Sagittarius Mines, Inc. executed a Deed of Absolute Saleof Shares of Stocks on 23 January 2001.

 After due consideration and evaluation of the financial and technical qualifications of SagittariusMines, Inc., the DENR Secretary approved the transfer of the Columbio FTAA from WMC

Philippines to Sagittarius Mines, Inc. in the assailed Order. According to said Order, pursuant toSection 66 of Department Administrative Order No. 96-40, as amended, Sagittarius Mines, Inc.meets the qualification requirements as Contractor-Transferee of FTAA No. 02-95-XI, and thatthe application for transfer of said FTAA went thru the procedure and other requirements setforth under the law.

 Aggrieved by the transfer of the Columbio FTAA in favor of Sagittarius Mines, Inc., petitionerfiled a Petition for Review of the Order of the DENR Secretary with the Office of the President.Petitioner assails the validity of the 18 December 2001 Order of the Secretary of the

Page 24: Envi Case Digests 3.1

7/24/2019 Envi Case Digests 3.1

http://slidepdf.com/reader/full/envi-case-digests-31 24/51

Department of Environment and Natural Resources (DENR) approving the application for andthe consequent registration of FTAA No. 02-95-XI from WMC Philippines to Sagittarius Mines,Inc.on the ground that: 1) it violates the constitutional right of Lepanto to due process; 2) itpreempts the resolution of very crucial legal issues pending with the regular courts; and 3) itblatantly violates Section 40 of the Mining Act.

In a Decision dated 23 July 2002, the Office of the President dismissed the petition

ISSUE:

WHETHER OR NOT the Philippine Mining Act of 1995, particularly Section 40 thereof requiringthe approval of the President of the assignment or transfer of financial or technical assistanceagreements should have a retroactive application to the Columbio FTAA.

HELD:

NO. Applying the above-cited law retroactively would contradict the established legaldoctrine that statutes are to be construed as having only a prospective operation unless the

contrary is expressly stated or necessarily implied from the language used in the law.

In the case at bar, there is an absence of either an express declaration or an implicationin the Philippine Mining Act of 1995 that the provisions of said law shall be made to applyretroactively, therefore, any section of said law must be made to apply only prospectively, inview of the rule that a statute ought not to receive a construction making it act retroactively,unless the words used are so clear, strong, and imperative that no other meaning can beannexed to them, or unless the intention of the legislature cannot be otherwise satisfied.

Page 25: Envi Case Digests 3.1

7/24/2019 Envi Case Digests 3.1

http://slidepdf.com/reader/full/envi-case-digests-31 25/51

12.

G.R. No. 85904 August 21, 1990TEODORO MEDRANA, petitioner,vs.OFFICE OF THE PRESIDENT and SUPREME AGGREGATES CORPORATION, respondents.

Facts:

This controversy, which began in 1979, relates to Mining Lease Contract ("MLC") No. V-754which the then Secretary of Agriculture and Natural Resources issued to private respondentSupreme Aggregates Corporation ("Supreme Aggregates") on 30 June 1969. This Mining LeaseContract has a lifetime of twenty-five (25) years and covers twenty-two (22) mining claims forvolcanic cinder, etc., situated in Calamba, Laguna Province and Sto. Tomas, BatangasProvince.

On 14 May 1976, within the period prescribed in Section 100 above, Supreme Aggregates filedwith the Bureau of Mines an Application to Avail of Rights and Privileges under P.D. No. 463

which application was required by Sections 100 and 101 of PD No.463 known as "The MineralResources Development Decree of 1974, from all claimowners and lessees desirous ofmaintaining their pre-existing rights under the regime inaugurated by that decree.

On 27 February 1979, the Director of the Bureau of Mines issued an order denying Supreme Aggregates' Application on the ground that Supreme Aggregates had failed to submit Affidavitsof Annual Work Obligations.

On 15 June 1979, the Director of Mines issued Quarry Temporary Permits ("QTPs") Nos. 85, 86and 87 to petitioner Teodoro Medrana. These permits covered areas within the territory leasedto Supreme Aggregates under MLC No. V-754.Fourteen days later, on 29 June 1979, Supreme Aggregates filed a petition with the Director of

Mines praying for reinstatement of its rejected Application to Avail of Rights and Privileges andfor cancellation of Medrana's QTPs Nos. 85, 86 and 87. Medrana filed an answer to thispetition.

 After investigation, the Director of Mines rendered a decision dated 13 March 1981 ordering thereinstatement of Supreme Aggregates' Application to Avail of Rights and Privileges and thecancellation of Medrana's QTPs Nos. 85, 86 and 87 since these covered areas within Supreme

 Aggregates' valid and subsisting MLC No. V-754.

On appeal by Medrana, the Ministry of Natural Resources reinstated Medrana's QTPs Nos. 85,86 and 87, and declared that Supreme Aggregates' MLC No. V-754 had lapsed. In so deciding,former Minister of Natural Resources Teodoro Q. Peña reasoned that the order of the Bureau of

Mines which had rejected Supreme Aggregates' Application to Avail of Rights and Privilegeshad already become final when Supreme Aggregates filed its petition for reinstatement of its

 Application on 29 June 1979, that is, one hundred and twenty (120) days after its receipt of theBureau of Mines order of denial. Former Minister Pena further held that the failure of Supreme

 Aggregates to file an Application to Avail of Rights and Privileges under P.D. No. 463 caused itsMining Lease Contract to lapse and opened the leased area to relocation. 1

Supreme Aggregates then filed an appeal with the Office of the President. The Office of thePresident, as already noted, in a decision dated 20 September 1988, reversed the decision of

Page 26: Envi Case Digests 3.1

7/24/2019 Envi Case Digests 3.1

http://slidepdf.com/reader/full/envi-case-digests-31 26/51

the Minister of Natural Resources and in essence held that the failure to submit Affidavits of Annual Work Obligations for two (2) consecutive years did not, by itself and standing alone,result in the automatic cancellation of MLC No. V-754.

In the instant Petition for Certiorari, petitioner Medrana submits that the Office of the Presidentacted with grave abuse of discretion, amounting to lack of jurisdiction, in reinstating Supreme

 Aggregates' MLC No. V-754 and cancelling petitioner's QTPs for the following reasons:

1. Private respondent Supreme Aggregates had abandoned its Mining Lease Contract by failingto comply with the mandatory requirements of Section 27 of P.D. No. 463.

2. The Bureau of Mines had declared the quarry covered by MLC No. V-754 as open forrelocation of claims by virtue of the cancellation of that MLC. Moreover, petitioner Medrana,being registered owner of much of the land covered by MLC No. V-754, claims a preferentialright to exploit the said quarry under Section 67 of P.D. No. 463.

3. Even before the issuance of QTPs Nos. 85, 86 and 87 to petitioner, the order of the Directorof Mines denying Supreme Aggregates' Application had already become final and executory five

(5) days from receipt of that order by private respondent Supreme Aggregates.

Issue:

1. Whether or not there is abandonment on the part of Supreme Aggregates.

No. Abandonment may be said to result where there is concurrence of two (2) elements: the firstbeing the intent to abandon a right or claim and the second being the external act by which thatintention is expressed and carried into effect. There must, moreover, be an actual, asdistinguished from a merely projected, relinquishment of a claim or right; otherwise the right orclaim is not vacated or waived so as to be susceptible of being appropriated by the next owner.

These two (2) requirements are clearly lacking in the case at bar. The Director of Mines andpublic respondent Office of the President had found that, in point of fact, private respondentSupreme Aggregates had performed its annual work obligations. Supreme Aggregates couldnot therefore be said to have intended to abandon its mining claim or lease, notwithstanding thefact that it had failed to submit the normal documentary proof of performance of annual workobligations that is, the Affidavit of Annual Work Obligations.

2. Whether or not petitioner Medrana, being registered owner of much of the land covered byMLC No. V-754, has a preferential right to exploit the said quarry under Section 67 of P.D. No.463.

No. It is true that under Section 67 of P.D. No. 463, petitioner Medrana, as registered owner of

the superficies of the land here involved, had a "preferential right to exploit the quarry resourcesfound therein". That right, however, was simply a preferential right, and that right was ineffectiveto dissolve the pre-existing or subsisting right of private respondent Supreme Aggregates. Theorder of 27 February 1979 of the Director of Mines, denying Supreme Aggregates' Application to

 Avail of Rights and Privileges, did not have, nor did it purport to have, the effect of cancelling ordeclaring the automatic abandonment of MLC No. V-754; as noted above, there simply was nolegal basis for so cancelling or declaring it as abandoned. It follows that the 27 February 1979order did not produce the effect of opening up the areas leased in MLC No. V-754 to location ornew acquisition of lights by petitioner Medrana, or by any other person for that matter. 5 It

Page 27: Envi Case Digests 3.1

7/24/2019 Envi Case Digests 3.1

http://slidepdf.com/reader/full/envi-case-digests-31 27/51

follows, furthermore, that petitioner's QTPs Nos. 85, 86 and 87 were not validly issued to beginwith and that the express cancellation of the QTPs decreed in the Director of Mines' decision of13 March 1981, which formally rectified his error by reinstating Supreme Aggregates'

 Application, was not even necessary being mere confirmatory of the juridical situation.

3. Whether or not the order of the Director of Mines denying Supreme Aggregates' Application

had already become final and executory five (5) days from receipt of that order by privaterespondent Supreme Aggregates.

No. We have already pointed out above that the denial order of the Director of Mines did nothave the effect of ipso factocancelling MLC No. V-754. It remains only therefore to determinewhether, as petitioner contends and as the former Minister of Natural Resources held, therejection order of the Director of Mines became final and executory upon expiration of five (5)days from receipt thereof by Supreme Aggregates.

We agree with the Office of the President that Section 5 of P.D. No. 309 has no application tothe case at bar. As its title clearly indicates, P.D. No. 309 applies only to cases involvingconflicting mining claims, that is to say, to orders or decisions issued in adversarial or litigated

proceedings involving mining claims with conflicting or overlapping boundaries.

4. Whether or not The President has commited grave abuse of discretion in its decision.

No. Even if petitioner had succeeded in showing that the Office of the President had indeedmisconstrued some provision of P.D. No. 463 as amended, or of the Rules and Regulationsimplementing P.D. No. 463, or of P.D. No. 309, such error would still be merely an error of lawor an error of judgment and certainly not a grave abuse of discretion or an act without or inexcess of jurisdiction correctible by certiorari.

Page 28: Envi Case Digests 3.1

7/24/2019 Envi Case Digests 3.1

http://slidepdf.com/reader/full/envi-case-digests-31 28/51

13.

G.R. No. 98332 January 16, 1995Miners Association of the Philippines v. Factoran

FACTS:

Executive Order Nos. 211 and 279 were issued by the then Pres. Aquino. EO No. 211prescribes the interim procedures in the processing and approval of applications for theexploration, development and utilization of minerals pursuant to Section 2, Article XII of the1987 Constitution. EO No. 279 authorizes the DENR Secretary to negotiate and conclude joint-venture, co-production, or production- sharing agreements for the exploration, development,and utilization of mineral resources.

The issuance and the impeding implementation by the DENR of Administrative OrderNos. 57 which declares that all existing mining leases or agreements which were granted afterthe effectivity of the 1987 Constitution…shall be converted into production-sharing agreementswithin one (1) year from the effectivity of these guidelines.‖ and Administrative Order No. 82

which provides that a failure to submit Letter of Intent and Mineral Production-Sharing Agreement within 2 years from the effectivity of the Department Administrative Order No. 57shall cause the abandonment of the mining, quarry, and sand and gravel claims, after theirrespective effectivity dates compelled the Miners Association of the Philippines, Inc., anorganization composed of mining prospectors and claim owners and claim holders, to file theinstant petition assailing their validity and constitutionality before this Court.

ISSUE:

 Are the two Department Administrative Orders valid

HELD:

Yes. Petitioner's insistence on the application of Presidential Decree No. 463, asamended, as the governing law on the acceptance and approval of declarations of location andall other kinds of applications for the exploration, development, and utilization of mineralresources pursuant to Executive Order No. 211, is erroneous. Presidential Decree No. 463, asamended, pertains to the old system of exploration, development and utilization of naturalresources through "license, concession or lease" which, however, has been disallowed by

 Article XII, Section 2 of the 1987 Constitution. By virtue of the said constitutional mandate andits implementing law, Executive Order No. 279 which superseded Executive Order No. 211, theprovisions dealing on "license, concession or lease" of mineral resources under PresidentialDecree No. 463, as amended, and other existing mining laws are deemed repealed and,therefore, ceased to operate as the governing law. In other words, in all other areas of

administration and management of mineral lands, the provisions of Presidential Decree No. 463,as amended, and other existing mining laws, still govern. Section 7 of Executive Order No. 279provides, thus:

Sec. 7. All provisions of Presidential Decree No. 463, as amended, other existing mining laws,and their implementing rules and regulations, or parts thereof, which are not inconsistent withthe provisions of this Executive Order, shall continue in force and effect.

Page 29: Envi Case Digests 3.1

7/24/2019 Envi Case Digests 3.1

http://slidepdf.com/reader/full/envi-case-digests-31 29/51

Well -settled is the rule, however, that regardless of the reservation clause, mining leases oragreements granted by the State, such as those granted pursuant to Executive Order No. 211referred to this petition, are subject to alterations through a reasonable exercise of the policepower of the State.

 Accordingly, the State, in the exercise of its police power in this regard, may not be precluded

by the constitutional restriction on non-impairment of contract from altering, modifying andamending the mining leases or agreements granted under Presidential Decree No. 463, asamended, pursuant to Executive Order No. 211. Police Power, being co-extensive with thenecessities of the case and the demands of public interest; extends to all the vital public needs.The passage of Executive Order No. 279 which superseded Executive Order No. 211 providedlegal basis for the DENR Secretary to carry into effect the mandate of Article XII, Section 2 ofthe 1987 Constitution.

Page 30: Envi Case Digests 3.1

7/24/2019 Envi Case Digests 3.1

http://slidepdf.com/reader/full/envi-case-digests-31 30/51

14.

G.R. No. 163509

PICOP RESOURCES, INC., vs. BASE METALS MINERAL RESOURCESCORPORATION and THE MINES ADJUDICATION BOARD

FACTS:

Central Mindanao Mining and Development Corporation (CMMCI for brevity) entered into aMines Operating Agreement (Agreement for brevity) with Banahaw Mining and DevelopmentCorporation (Banahaw Mining for brevity) whereby the latter agreed to act as Mine Operator forthe exploration, development, and eventual commercial operation of CMMCI‘s eighteen (18)mining claims located in Agusan del Sur.

Pursuant to the terms of the Agreement, Banahaw Mining filed applications for Mining LeaseContracts over the mining claims with the Bureau of Mines. So that Banahaw Mining was issueda Mines Temporary Permit authorizing it to extract and dispose of precious minerals found

within its mining claims. Upon its expiration, the temporary permit was subsequently renewedthrice by the Bureau of Mines, the last being on June 28, 1991.

Since a portion of Banahaw Mining‘s mining claims was located in petitioner PICOP‘s loggingconcession in Agusan del Sur, Banahaw Mining and petitioner PICOP entered into aMemorandum of Agreement, whereby, in mutual recognition of each other‘s right to the areaconcerned, petitioner PICOP allowed Banahaw Mining an access/right of way to its miningclaims. Banahaw Mining converted its mining claims to applications for Mineral ProductionSharing Agreements (MPSA for brevity).

While the MPSA were pending, Banahaw Mining, on December 18, 1996, decided to sell/assignits rights and interests over thirty-seven (37) mining claims in favor of private respondent Base

Metals Mineral Resources Corporation (Base Metals for brevity). The transfer included miningclaims held by Banahaw Mining in its own right as claim owner, as well as those covered by itsmining operating agreement with CMMCI.

Upon being informed of the development, CMMCI, as claim owner, immediately approved theassignment made by Banahaw Mining in favor of private respondent Base Metals, therebyrecognizing private respondent Base Metals as the new operator of its claims.

On March 10, 1997, private respondent Base Metals amended Banahaw Mining‘s pendingMPSA applications with the Bureau of Mines to substitute itself as applicant and to submitadditional documents in support of the application. Area clearances from the DENR RegionalDirector and Superintendent of the Agusan Marsh and Wildlife Sanctuary were submitted, as

required.

On October 7, 1997, private respondent Base Metals‘ amended MPSA applications werepublished in accordance with the requirements of the Mining Act of 1995.

On November 18, 1997, petitioner PICOP filed with the Mines Geo-Sciences Bureau (MGB),Caraga Regional Office No. XIII an Adverse Claim and/or Opposition to private respondentBase Metals‘ application. After the submission of their respective position paper, the Panel

 Arbitrator issued an Order disapproving private respondent Base Metals‘ MPSA on the reasons

Page 31: Envi Case Digests 3.1

7/24/2019 Envi Case Digests 3.1

http://slidepdf.com/reader/full/envi-case-digests-31 31/51

that adverse claim was filed on time, that the granting of the MPSA application on area subjectof an IFMA or PTLA which is covered by a Presidential Warranty, the panel believes it cannot,unless the grantee consents thereto, without the grantee‘s consent, the area is consideredclosed to mining location (sec. 19) (b) (No. 2), DAO No. 96-40) and that the mining location inforest or timberland is allowed only if such forest or timberland is not leased by the governmentto a qualified person or entity and if it is leased the consent of the lessor is necessary, in

addition to the area clearance to be issued by the agency concerned before it is subjected tomining operation.

Plantation is considered closed to mining locations because it is off tangent to mining. Both areextremes. They can not exist at the same time. The other must necessarily stop before the otheroperate.

Private respondent Base Metals filed a Notice of Appeal with public respondent MAB, the latterrendered the assailed decision setting aside the Panel Arbitrator‘s order. The Court of Appealsupheld the decision of the MAB.

Hence this petition.

PICOP presents the following issues: (1) the 2,756 hectares subject of Base Metals‘ MPSA areclosed to mining operations except upon PICOP‘s written consent pursuant to existing laws,rules and regulations and by virtue of the Presidential Warranty; (2) its Presidential Warranty isprotected by the non-impairment clause of the Constitution; and (3) it does not raise new issuesin its petition.

PICOP asserts that its concession areas are closed to mining operations as these are within the Agusan-Surigao-Davao forest reserve established under Proclamation No. 369 of then Gov.Gen. Dwight Davis. The area is allegedly also part of permanent forest established underRepublic Act No. 3092 (RA 3092), and overlaps the wilderness area where mining applicationsare expressly prohibited under RA 7586. Hence, the area is closed to mining operations under

Sec. 19(f) of RA 7942.

ISSUE:

Whether or not the area covered by Base Metals‘ MPSA is, by law, closed to miningactivities

Whether or not the Presidential Warranty is a contract protected by the non-impairmentclause of the 1987 Constitution.

HELD:

 Anent the first issue, the Court ruled that the area covered by Base Metals‘ MPSA is, bylaw, not closed to mining activities.

There is no evidence in this case that the area covered by Base Metals‘ MPSA has beenproclaimed as watershed forest reserves.

Even granting that the area covered by the MPSA is part of the Agusan-Davao-SurigaoForest Reserve, such does not necessarily signify that the area is absolutely closed to miningactivities. Contrary to PICOP‘s obvious misreading of our decision in Apex Mining Co., Inc. v.

Page 32: Envi Case Digests 3.1

7/24/2019 Envi Case Digests 3.1

http://slidepdf.com/reader/full/envi-case-digests-31 32/51

Garcia, supra, to the effect that mineral agreements are not allowed in the forest reserveestablished under Proclamation 369, the Court in that case actually ruled that pursuant to PD463 as amended by PD 1385, one can acquire mining rights within forest reserves, such as the

 Agusan-Davao-Surigao Forest Reserve, by initially applying for a permit to prospect with theBureau of Forest and Development and subsequently for a permit to explore with the Bureau ofMines and Geosciences.

Moreover, Sec. 18 RA 7942 allows mining even in timberland or forestty subject toexisting rights and reservations. Similarly, Sec. 47 of PD 705 permits mining operations inforest lands which include the public forest, the permanent forest or forest reserves, and forestreservations

With regard to the second issue, the Court do not subscribe to PICOP‘s argument thatthe Presidential Warranty dated September 25, 1968 is a contract protected by the non-impairment clause of the 1987 Constitution. An examination of the Presidential Warranty atonce reveals that it simply reassures PICOP of the government‘s commitment to uphold theterms and conditions of its timber license and guarantees PICOP‘s peaceful and adequatepossession and enjoyment of the areas which are the basic sources of raw materials for its

wood processing complex. The warranty covers only the right to cut, collect, and remove timberin its concession area, and does not extend to the utilization of other resources, such as mineralresources, occurring within the concession.

The Presidential Warranty cannot be considered a contract distinct from PTLA No. 47and IFMA No. 35. It is merely a collateral undertaking which cannot amplify PICOP‘s rightsunder its timber license. Since timber licenses are not contracts, the non-impairment clausecannot be invoked.

Page 33: Envi Case Digests 3.1

7/24/2019 Envi Case Digests 3.1

http://slidepdf.com/reader/full/envi-case-digests-31 33/51

15.

REPUBLIC OF THE PHILIPPINES (DIRECTOR OF FOREST DEVELOPMENT), petitioner,vs. HON. COURT OF APPEALS (THIRD DIVISION) and JOSE Y. DE LA ROSA,respondents1988-04-15 | G.R. No. L-43938

FACTS:

• These cases arose from the application for registration of a parcel of land filed o n February 11,1965, by Jose de la Rosa on his own behalf and on behalf of his three children, Victoria,Benjamin and Eduardo.

• The land, situated in Tuding, Itogon, Benguet Province, was divided into 9 lots..  

• According to the application, Lots 1-5 were sold to Jose de la Rosa and Lots 6-9 to his childrenby Mamaya Balbalio and Jaime Alberto, respectively, in 1964.

• The application was separately opposed by Benguet Consolidated, Inc. as to Lots 1-5, AtokBig Wedge Corporation, as to portions of Lots 1-5 and all of Lots 6-9, and by the Republic of thePhilippines, through the Bureau of Forestry Development, as to Lots 1-9.

• In support of the application, both Balbalio and Alberto testified that they had acquired thesubject land by virtue of prescription. Balbalio claimed to have received Lots 1-5 from her fathershortly after the Liberation. She testified she was born in the land, which was possessed by herparents under claim of ownership.

• Alberto said he received Lots 6-9 in 1961 from his mother, Bella Alberto, who declared that theland was planted by Jaime and his predecessors-in-interest to bananas, avocado, nangka andcamote, and was enclosed with a barbed-wire fence. She was corroborated by Felix Marcos, 67

years old at the time, who recalled the earlier possession of the land by Alberto's father.

• Balbalio presented her tax declaration in 1956 and the realty tax receipts from that year to1964, 6 Alberto his tax declaration in 1961 and the realty tax receipts from that year to 1964.

• Benguet opposed on the ground that the June Bug mineral claim covering Lots 1-5 was sold toit on September 22, 1934, by the successors-in-interest of James Kelly, who located the claim inSeptember 1909 and recorded it on October 14, 1909.

• From the date of its purchase, Benguet had been in actual, continuous and exclusivepossession of the land in concept of owner, as evidenced by its construction of adits, itsaffidavits of annual assessment, its geological mappings, geological samplings and trench side

cuts, and its payment of taxes on the land.

• The Bureau of Forestry Development also interposed its objection, arguing that the landsought to be registered was covered by the Central Cordillera Forest Reserve underProclamation No. 217 dated February 16, 1829. Moreover, by reason of its nature, it was notsubject to alienation under the Constitutions of 1935 and 1973.

• The trial court denied the application, holding that the applicants had failed to prove their claimof possession and ownership of the land sought to be registered.

Page 34: Envi Case Digests 3.1

7/24/2019 Envi Case Digests 3.1

http://slidepdf.com/reader/full/envi-case-digests-31 34/51

 • The applicants appealed to the CA, which reversed the trial court and recognized the claims ofthe applicant, but subject to the rights of Benguet and Atok respecting their mining claims.

• In other words, the Court of  Appeals affirmed the surface rights of the de la Rosas over theland while at the same time reserving the sub-surface rights of Benguet and Atok by virtue of

their mining claims.

• Both Benguet and Atok have appealed to the Supreme Court, invoking their superior right ofownership. The Republic has filed its own petition for review and reiterates its argument thatneither the private respondents nor the two mining companies have any valid claim to the landbecause it is not alienable and registerable.

ISSUE:

• WON Both Benguet and Atok have exclusive right of ownership over the land. 

• WON neither the private respondents nor the two mining companies have any valid claim to

the land because it is not alienable and registrable.

RULING:

• SC ruled that Benguet and Atok have exclusive rights to the property in question by virtue oftheir respective mining claims which they validly acquired before the Constitution of 1935prohibited the alienation of all lands of the public domain except agricultural lands, subject tovested rights existing at the time of its adoption. The land was not and could not have beentransferred to the private respondents by virtue of acquisitive prescription, nor could its use beshared simultaneously by them and the mining companies for agricultural and mineral purposes.

The flaw in the reasoning of the respondent court is in supposing that the rights over the land

could be used for both mining and non-mining purposes simultaneously. The correctinterpretation is that once minerals are discovered in the land, whatever the use to which it isbeing devoted at the time, such use may be discontinued by the State to enable it to extract theminerals therein in the exercise of its sovereign prerogative. The land is thus converted tomineral land and may not be used by any private party, including the registered owner thereof,for any other purpose that will impede the mining operations to be undertaken therein. For theloss sustained by such owner, he is of course entitled to just compensation under the MiningLaws or in appropriate expropriation proceedings.

• It is true that the subject property was considered forest land and included in the CentralCordillera Forest Reserve, but this did not impair the rights already vested in Benguet and Atokat that time. The Court of Appeals correctly declared that:

"There is no question that the 9 lots applied for are within the June Bug mineral claims ofBenguet and the 'Fredia and Emma' mineral claims of Atok. The June Bug mineral claim ofplaintiff Benguet was one of the 16 mining claims of James E. Kelly, an American and mininglocator. He filed his declaration of the location of the June Bug mineral and the same wasrecorded in the Mining Recorder's Office on October 14, 1909. All of the Kelly claims hadsubsequently been acquired by Benguet Consolidated, Inc. Benguet's evidence is that it hadmade improvements on the June Bug mineral claim consisting of mine tunnels prior to 1935. Ithad submitted the required affidavit of annual assessment. After World War II, Benguet

Page 35: Envi Case Digests 3.1

7/24/2019 Envi Case Digests 3.1

http://slidepdf.com/reader/full/envi-case-digests-31 35/51

introduced improvements on mineral claim June Bug, and also conducted geological mappings,geological sampling and trench side cuts. In 1948, Benguet redeclared the 'June Bug' fortaxation and had religiously paid the taxes.

"The Emma and Fredia claims were two of the several claims of Harrison registered in 1931,and which Atok representatives acquired. Portions of Lots 1 to 5 and all of Lots 6 to 9 are within

the Emma and Fredia mineral claims of Atok Big Wedge Mining Company. "The June Bug mineral claim of Benguet and the Fredia and Emma mineral claims of Atokhaving been perfected prior to the approval of the Constitution of the Philippines of 1935, theywere removed from the public domain and had become private properties of Benguet and Atok.We agree likewise with the oppositors that having complied with all the requirements of themining laws, the claims were removed from the public domain, and not even the government ofthe Philippines can take away this right from them. The reason is obvious. Having become theprivate properties of the oppositors, they cannot be deprived thereof without due process of law.

Page 36: Envi Case Digests 3.1

7/24/2019 Envi Case Digests 3.1

http://slidepdf.com/reader/full/envi-case-digests-31 36/51

18.

G.R. No. 137174 July 10, 2000REPUBLIC OF THE PHILIPPINES, Represented by the POLLUTION ADJUDICATIONBOARD (DENR) vs. MARCOPPER MINING CORPORATION

FACTS:

Respondent MMC was issued a temporary permit to operate a tailings sea disposal system. Inthe meantime, the National Pollution Control Commission (NPCC) was abolished by EO No.192 dated June 10, 1987, and its powers and functions were integrated into the EnvironmentalManagement Bureau and into the Pollution Adjudication Board (PAB).

On April 11, 1988, the DENR Secretary, in his capacity as Chairman of the PAB, issued anOrder directing MMC to "cease and desist from discharging mine tailings into Calancan Bay."This was appealed by the MMC with the Office of the President (OP).

In line with the directive from the OP, the Calancan Bay Rehabilitation Project (CBRP) was

created, and MMC remitted the amount of P30,000.00 a day, starting from May 13, 1988 to theEcology Trust Fund (ETF) thereof. However, on June 30, 1991, MMC stopped discharging itstailings in the Bay, hence, it likewise ceased from making further deposits to the ETF.

The PAB sought for the enforcement of the order issued by the OP, however, the CA acted onMarcopper‘s petition and ordered the PAB to refrain and desist from enforcing aforesaid Order.Hence, the instant petition.

ISSUE:

The Court of Appeals erred in ruling that Republic Act No. 7942 repealed the provisions ofRepublic Act No. 3931, as amended by Presidential Decree No. 984, with respect to the power

and function of petitioner Pollution Adjudication Board to issue, renew or deny permits for thedischarge of the mine tailings.

HELD:

The SC held that the CA erred in ruling that the PAB had no authority to issue the Order fromthe ruling of the Court of Appeals that the PAB has been divested of authority to act onpollution-related matters in mining operations is anchored on the provisions of RA 7942(Philippine Mining Act of 1995). However, Section 19 of EO 192 vested the PAB with thespecific power to adjudicate pollution cases in general. Sec. 2, par. (a) of PD 984 defines theterm "pollution" as referring to any alteration of the physical, chemical and biological propertiesof any water, air and/or land resources of the Philippines , or any discharge thereto of any liquid,

gaseous or solid wastes as will or is likely to create a harmful environment.

On the other hand, the authority of the mines regional director is complementary to that of thePAB. While the mines regional director has express administrative and regulatory powers overmining operations and installations, it has no adjudicative powers over complaints for violation ofpollution control statutes and regulations. Contrary to the ruling of the CA, RA 7942 does notvest quasi-judicial powers in the Mines Regional Director. The authority is vested and remainswith the PAB. Neither was such authority conferred upon the Panel of Arbitrators and the Mines

 Adjudication Board which were created by the said law. The scope of authority of the Panel of

Page 37: Envi Case Digests 3.1

7/24/2019 Envi Case Digests 3.1

http://slidepdf.com/reader/full/envi-case-digests-31 37/51

 Arbitrators and the Mines Adjudication Board conferred by RA 7942 clearly exclude adjudicativeresponsibility over pollution cases.

Page 38: Envi Case Digests 3.1

7/24/2019 Envi Case Digests 3.1

http://slidepdf.com/reader/full/envi-case-digests-31 38/51

19.

PEOPLE v. ROSEMOORG.R. No. 149927March 30, 2004

FACTS:

Rosemoor Mining And Development Corporation Corporation (Rosemoor), after having beengranted permission to prospect for marble deposits in the mountains of Biak-na-Bato, Bulacan,succeeded in discovering marble deposits of high quality and in commercial quantities. Theyapplied with the Bureau of Mines (now Mines and Geosciences Bureau), for the issuance of thecorresponding license to exploit said marble deposits, which was issued to them, giving themthe right to quarry 330 hectares of land. After Ernesto Maceda was appointed Minister of DENR,he cancelled Rosemoor‘s license. Rosemoor filed for injunctive relief from the RTC, which ruledin their favor.

The RTC said that Rosewood‘s respondents‘ license had already ripened into a property right,which was protected under the due process clause, and such right was supposedly violatedwhen the license was unjustifiably cancelled without notice and hearing. Petitioners aver thatthe license contravenes PD 463 because it exceeds the maximum area that may be granted toa Licensee for quarrying (100 hectares), which renders the license void. The CA sustained theRTC decision because the license was embraced by four (4) separate applications, and that the100 hectare limitation was superseded by RA 7942. The CA also said that Proclamation 84,which confirmed the cancellation of the license, impaired the non-impairment clause ofcontracts, a bill of attainder and an ex post facto law.

ISSUE:

W/N Rosemoor‘s license was validly cancelled. – YES.

RATIO:

Validity of License

PD 463, as amended, pertained to the old system of exploration, development andutilization of natural resources through licenses, concessions or leases, but was omitted in the1987 Constitution as it was deemed violative of its provisions. This was replaced by RA 7942 orthe Philippine Mining Act of 1995 repealed or amended all laws, executive orders, presidentialdecrees, rules and regulations -- or parts thereof -- that are inconsistent with it.

While RA 7942 has expressly repealed provisions of mining laws that are inconsistentwith its own, it nonetheless respects previously issued valid and existing licenses. In this case,the terms of Rosemoor‘s license was subject to PD 463, the existing law when it was granted.

 And under such law, it is clear that a license should only cover 100 hectares without exceptionsor consideration to the number of applications. The intent of the law would be brazenlycircumvented by ruling that a license may cover an area exceeding the maximum by the mereexpediency of filing several applications. Such ruling would indirectly permit an act that isdirectly prohibited by the law.

Page 39: Envi Case Digests 3.1

7/24/2019 Envi Case Digests 3.1

http://slidepdf.com/reader/full/envi-case-digests-31 39/51

Validity of Proclamation No. 84

Rosemoor‘s license may be revoked or rescinded by executive action when the nationalinterest so requires, because it is not a contract, property or a property right protected by thedue process clause of the Constitution. This condition to the license was acknowledged byRosemoor in its permit. Moreover, granting that Rosemoor‘ license is valid, it can still be validly

revoked by the State in the exercise of police power. The exercise of such power throughProclamation No. 84 is clearly in accord with the regalia doctrine which reserves to the Stateownership of all natural resources.

Proclamation 84 does not impair the non-impairment clause because the license is not acontract. Even if the license were, it is settled that provisions of existing laws and a reservationof police power are deemed read into it, because it concerns a subject impressed with publicwelfare. PN 84 is also not a bill of attainder because the declaration that the license was void isnot a punishment. It is also not an ex post facto law because the proclamation does not fallunder any of the enumerated categories of an ex post facto law. And an ex post facto law islimited in its scope only to matters criminal in nature.

Page 40: Envi Case Digests 3.1

7/24/2019 Envi Case Digests 3.1

http://slidepdf.com/reader/full/envi-case-digests-31 40/51

20.

[G.R. No. 135190. April 3, 2002]SOUTHEAST MINDANAO GOLD MINING CORPORATION, petitioner, vs. BALITE PORTALMINING COOPERATIVE and others similarly situated; and THE HONORABLE ANTONIOCERILLES, in his capacity as Secretary of the Department of Environment and Natural

Resources (DENR), PROVINCIAL MINING REGULATORY BOARD OF DAVAO (PMRB-Davao), respondents.

FACTS:

The instant case involves a rich tract of mineral land situated in the Agusan-Davao-SurigaoForest Reserve known as the ―Diwalwal Gold Rush Area.‖ the land has been embroiled incontroversy (DIWALWAL Conflict) since the mid-80‘s due to the scramble over gold depositsfound within its bowels.

On June 24, 1997, the DENR Secretary issued Memorandum Order No. 97-03 which provided,among others, that:

―The DENR shall study thoroughly and exhaustively the option of direct state utilization of themineral resources in the Diwalwal Gold-Rush Area. Such study shall include, but shall not belimited to, studying and weighing the feasibility of entering into management agreements oroperating agreements, or both, with the appropriate government instrumentalities or privateentities, or both, in carrying out the declared policy of rationalizing the mining operations in theDiwalwal Gold Rush Area; such agreements shall include provisions for profit-sharing betweenthe state and the said parties, including profit-sharing arrangements with small-scale miners, aswell as the payment of royalties to indigenous cultural communities, among others. TheUndersecretary for Field Operations, as well as the Undersecretary for Legal and Legislative

 Affairs and Attached Agencies, and the Director of the Mines and Geo-sciences Bureau arehereby ordered to undertake such studies.‖ x x x 

On July 16, 1997, petitioner filed a special civil action for certiorari, prohibition and mandamusbefore the Court of Appeals. It prayed for the nullification of Memorandum Order No. 97-03.On March 19, 1998, the Court of Appeals, dismissed the petition. It ruled that the DENRSecretary did not abuse his discretion in issuing Memorandum Order No. 97-03 since the samewas merely a directive to conduct studies on the various options available to the government forsolving the Diwalwal conflict. The assailed memorandum did not conclusively adopt ―directstate utilization‖ as official government policy on the matter, but was simply a manifestation ofthe DENR‘s intent to consider it as one of its options, after determining its feas ibility throughstudies. MO 97-03 was only the initial step in the ladder of administrative process and did not,as yet, fix any obligation, legal relationship or right.

Petitioner filed a motion for reconsideration, which was denied for lack of merit. Hence thispetition.

ISSUE:

WON CA erred when it concluded that the assailed memorandum order did not adopt the ―directstate utilization scheme‖ in resolving the Diwalwal Conflict. 

Page 41: Envi Case Digests 3.1

7/24/2019 Envi Case Digests 3.1

http://slidepdf.com/reader/full/envi-case-digests-31 41/51

HELD:

No since the challenged MO 97-03 did not conclusively adopt ―direct state utilization‖ as a policyin resolving the Diwalwal dispute. The terms of the memorandum clearly indicate that what wasdirected thereunder was merely a study of this option and nothing else. Contrary to petitioner‘scontention, it did not grant any management/operating or profit-sharing agreement to small-

scale miners or to any party, for that matter, but simply instructed the DENR officials concernedto undertake studies to determine its feasibility.

 Additionally, there can be no valid opposition raised against a mere study of an alternativewhich the State, through the DENR, is authorized to undertake in the first place. Worth noting is

 Article XII, Section 2, of the 1987 Constitution , which specifically provides:

SEC. 2. All lands of the public domain, waters, minerals, coal, petroleum, and other mineral oils,all forces of potential energy, fisheries, forests or timber, wildlife, flora and fauna, and othernatural resources are owned by the State. With the exception of agricultural lands, all othernatural resources shall not be alienated. The exploration, development, and utilization ofnatural resources shall be under the full control and supervision of the State. The State may

directly undertake such activities, or it may enter into co-production, joint venture, or production-sharing agreements with Filipino citizens, or corporations or associations at least sixty percentum of whose capital is owned by such citizens. Such agreements may be for a period notexceeding twenty-five years, renewable for not more than twenty-five years, and under suchterms and conditions as may be provided by law. In cases of water rights for irrigation, watersupply, fisheries, or industrial uses other than the development of water power, beneficial usemay be the measure and limit of the grant. (Underscoring ours)

Likewise, Section 4, Chapter II of the Philippine Mining Act of 1995 states:

SEC. 4. Ownership of Mineral Resources. - Mineral Resources are owned by the State and theexploration, development, utilization, and processing thereof shall be under its full control and

supervision. The State may directly undertake such activities or it may enter into mineralagreements with contractors. (Underscoring ours)

Thus, the State may pursue the constitutional policy of full control and supervision of theexploration, development and utilization of the country‘s natural mineral resources, by eitherdirectly undertaking the same or by entering into agreements with qualified entities. The DENRSecretary acted within his authority when he ordered a study of the first option, which may beundertaken consistently in accordance with the constitutional policy enunciated above.Obviously, the State may not be precluded from considering a direct takeover of the mines, if itis the only plausible remedy in sight to the gnawing complexities generated by the gold rush. Asimplied earlier, the State need be guided only by the demands of public interest in settling forthis option, as well as its material and logistic feasibility.

In this regard, petitioner‘s imputation of bad faith on the part of the DENR Secretary when thelatter issued MO 97-03 is not well-taken. Hence petition denied.

Page 42: Envi Case Digests 3.1

7/24/2019 Envi Case Digests 3.1

http://slidepdf.com/reader/full/envi-case-digests-31 42/51

21.

Case: STANDARD MINERAL PRODUCTS, INC. vs THE HON. COURT OF APPEALSGr No. L-43277.Date: April 26, 1990Ponente: MELENCIO-HERRERA, J.:

Facts:

 A Petition for Review on Certiorari of the Decision of Respondent Appellate Court, affirming the judgment of the former Court of First Instance of Rizal denying surface rights for miningpurposes to Petitioner.

Petitioner-Appellant Standard Mineral Products, Inc. (SMPI, for short) claims that it is the locatorof placer mining claims "Celia IV" and "Celia VI" containing limestone in Kaysipot, Antipolo,Rizal, which were duly registered in the Office of the Mining Recorder of Rizal Theaforementioned mining claims cover about fifteen (15) hectares of the one hundred-twenty (120)hectares of land registered in the name of Respondent-Appellee, Rufino Deeunhong

 After locating the claims, SMPI applied for a mining lease from the Bureau of Mines. TheLandowners opposed the application on the ground that SMPI had entered their land and filedits mining lease application without their permission.

SMPI brought an action in the Court of First Instance of Rizal against Respondents-Appelleespraying that it be granted surface rights for mining purposes.

The Landowners traversed the Complaint, by averring that SMPI is not entitled to the reliefdemanded because the prospecting was accomplished without previously securing theLandowner‘s written permission as surface owners as required by Section 27 of the Mining Act(Commonwealth Act No. 137, as amended)

Trial Court, finding that the mineral claims were not located in accordance with law dismissedthe complaint and, on the counterclaim, sentenced SMPI to pay to Deeunhong and theTanjuatcos actual damages in the sum of P50,000.00 each, attorney‘s fees of P5,000.00 andcosts. The Appellate Court 1 affirmed that Decision with the sole modification that temperate ormoderate damages (not actual damages) of P25,000.00 each were awarded instead.

In another Resolution the Appellate Court likewise denied, for being devoid of legal interest, thePetition for Intervention filed by the Republic of the Philippines through the Solicitor General.

 After SMPI elevated the case to this Court for review on Certiorari, the Republic reiterated itsPetition for Intervention, which we granted in the Resolution.

Issue:

WON SMPI is entitled to such surface rights

Held:

SMPI is not entitled to said surface rights as it failed to comply with the requisite of prior writtenpermission by the Landowners before entering the private land in question.

Page 43: Envi Case Digests 3.1

7/24/2019 Envi Case Digests 3.1

http://slidepdf.com/reader/full/envi-case-digests-31 43/51

Section 27 of the Mining Act explicitly provides:

"Section 27. Before entering private lands the prospector shall first apply in writing for writtenpermission of the private owner, claimant, or holder thereof, and in case of refusal by suchprivate owner, claimant, or holder to grant such permission, or in case of disagreement as to theamount of compensation to be paid for such privilege of prospecting therein, the amount of such

compensation shall be fixed by agreement among the prospector, the Director of the Bureau ofMines and the surface owner, and in case of their failure to unanimously agree as to the amountof compensation, all questions at issue shall be determined by the Court of First Instance of theprovince in which said lands are situated in an action instituted for the purpose by theprospector, or his principal: Provided, however, that the prospector, or his principal upondepositing with the court the sum considered jointly by him and the Director of the Bureau ofMines and the court to be Just compensation for the damages resulting from such prospecting,shall be permitted to enter upon, and locate the said land without such written permissionpending final adjudication of the amount of such compensation; and in such case theprospector, or his principal, shall have a prior right as against the world, from the date of hisapplication. The court in its final judgment, besides determining the correspondingcompensation of the damages which may be caused by the prospecting, shall make a

pronouncement as to the value and the reasonable rental for the occupation and utilizationthereof for mining purposes in case the prospector decides to locate and exploit the mineralsfound therein."

The purpose of the law is obvious, which is, to prevent trespass on private property. Theimportance of the written permission of the owner of private land is also apparent from the formsprescribed by the Bureau of Mines for the declaration of location of a mining claim which requirethe locator to state that the landowner has granted written permission for the prospecting andlocation of the mining claim if the latter is located on private property.

Section 27 is inapplicable as it never entered the land for the purpose of "prospecting" butalready for "locating" a mining claim inasmuch as the limestone deposits were prominently

exposed and spread visibly and recognizably on the surface of the land such that there was noneed of "entering" the land.‘ In finding the same to be without merit, suffice it to state that"entering" has to be precede "prospecting" ; "prospecting" necessarily precedes "discovery" ;and a valid "discovery" is essential for the "location" of a mining claim.

"Section 26 of the Mining Act provides that prospecting shall be carried on ‗in accordance withthe provisions of this Act.‘ As appellant‘s prospecting was done in violation of the law, it was anillegal act and the subsequent location of the mining claims was also illegal and null and void.For the Mining Act regards a valid discovery as that which gives the prospector the right tolocate a mining claim (Sections 29 and 30), and the validity of a location depends uponcompliance with the law.

‗It is clear, of course that the validity of a location depends upon compliance with the statutes.The law requires that the locator shall act in good faith, and it will not countenance a trespass asthe basis of a mining right‘ 

SMPI‘s suggestion that the remedy provided in Section 67 of the Mining Act be applied to it isalso unacceptable. It is evident that the foregoing speaks of lease of a mining claim to whichSMPI would neither be entitled for failure to comply with the provisions of the Mining Act and toaccompany its application for lease with a written authority of the Landowners. In fact, SMPI leftthe space provided for the same in its application blank. For the same reasons, authority cannot

Page 44: Envi Case Digests 3.1

7/24/2019 Envi Case Digests 3.1

http://slidepdf.com/reader/full/envi-case-digests-31 44/51

be granted by the Court, nor can rental be fixed, compliance with the terms of this Act being anindispensable prerequisite.

 A condition sine qua non is that the prospecting, exploration, discovery and location must bedone in accordance with the law. As it is, SMPI‘s rights to use and exploit the mineral resourcesdiscovered and located never matured because of its omission to comply with a condition

precedent. To allow SMPI its claim for surface rights and right of way would be to countenanceillegal trespass into private property.

WHEREFORE, with the sole modification as to the award of temperate damages, which arehereby reduced as indicated, the judgment under review is hereby affirmed in all other respects

Page 45: Envi Case Digests 3.1

7/24/2019 Envi Case Digests 3.1

http://slidepdf.com/reader/full/envi-case-digests-31 45/51

22.

G.R. No. 163101

BENGUET CORPORATION vs. DEPARTMENT OF ENVIRONMENT AND NATURALRESOURCES-MINES ADJUDICATION BOARD and J.G. REALTY AND MINING

CORPORATION

FACTS:

Benguet and J.G. Realty entered into a Royalty Agreement with Option to Purchase (RAWOP) ,wherein J.G. Realty was acknowledged as the owner of four mining claims with a total area of288.8656 hectares. The parties also executed a Supplemental Agreement. The mining claimswere covered by Mineral Production Sharing Agreement (MPSA) Application No. APSA-V-0009

 jointly filed by J.G. Realty as claim-owner and Benguet as operator.

 After some time, the Executive Vice-President of Benguet, Antonio N. Tachuling, issued a letterinforming J.G. Realty of its intention to develop the mining claims. However, J.G. Realty,

through its President, Johnny L. Tan, then sent a letter to the President of Benguet informing thelatter that it was terminating the RAWOP. The latter alleged that petitioner violated some of theprovisions of the RAWOP, specifically on non-payment of royalties and non-fulfillment ofobligations stipulated therein.

J.G. Realty filed a Petition for Declaration of Nullity/Cancellation of the RAWOP. POA issued aDecision, cancelling the RAWOP and its Supplemental Agreement. BENGUET wassubsequently excluded from the joint MPSA Application over the mineral claims. SubsequentMR was denied. Said decision was upheld by DENR-MAB.

Hence this instant petition.

ISSUE:

Whether or not the filing of the petition with the Supreme Court is proper.

HELD:

NO. the instant petition can be denied outright as Benguet resorted to an improper Remedy.

The last paragraph of Section 79 of Republic Act No. (RA) 7942 or the ―Philippine Mining Act of1995‖ states, ―A petition for review by certiorari and question of law may be filed by theaggrieved party with the Supreme Court within thirty (30) days from receipt of the order ordecision of the [MAB].‖ 

The Revised Rules of Civil Procedure included Rule 43 to provide a uniform rule on appealsfrom quasi-judicial agencies. Under the rule, appeals from their judgments and final orders arenow required to be brought to the CA on a verified petition for review. A quasi-judicial agency orbody has been defined as an organ of government, other than a court or legislature, whichaffects the rights of private parties through either adjudication or rule-making. MAB falls underthis definition; hence, it is no different from the other quasi-judicial bodies enumerated underRule 43. Besides, the introductory words in Section 1 of Circular No. 1-91 ––―among theseagencies are‖––indicate that the enumeration is not exclusive or conclusive and acknowledge

Page 46: Envi Case Digests 3.1

7/24/2019 Envi Case Digests 3.1

http://slidepdf.com/reader/full/envi-case-digests-31 46/51

the existence of other quasi-judicial agencies which, though not expressly listed, should bedeemed included therein.

The judicial policy of observing the hierarchy of courts dictates that direct resort fromadministrative agencies to this Court will not be entertained, unless the redress desired cannotbe obtained from the appropriate lower tribunals, or unless exceptional and compelling

circumstances justify availment of a remedy falling within and calling for the exercise of ourprimary jurisdiction.

Thus Benguet should have filed the appeal with the CA.

Petitioner having failed to properly appeal to the CA under Rule 43, the decision of the MAB hasbecome final and executory. On this ground alone, the instant petition must be denied.

Page 47: Envi Case Digests 3.1

7/24/2019 Envi Case Digests 3.1

http://slidepdf.com/reader/full/envi-case-digests-31 47/51

24.

G.R. No. 139548. December 22, 2000

MARCOPPER MINING CORPORATION vs. ALBERTO G. BUMOLO et al.,

FACTS:

MARCOPPER MINING CORPORATION registered its mining claims in Pao, Kasibu,Nueva Vizcaya with the DENR from February 02,1982 to October 12, 1982. Privaterespondents Alberto G. Bumolo and others registered their mining claims in the same area from28 July 1981 to 22 September 1988, which claims were subsequently converted into MineralProduction Sharing Agreements (MPSA).

On March 12, 1982 petitioner entered into Option Agreements over the mining. Underthe Agreements, petitioner was granted the exclusive and irrevocable right to explore the miningclaims for three (3) years with provision for extension.

On December 23, 1982 and March 26, 1987 petitioner filed Prospecting Permit Applications (PPA) with the Bureau of Forest Development, DENR, on the alleged ground that aportion of the area covered by the mining claims was within the Magat River Forest Reservationunder Proc. 573 of June 26, 1969 and with DAR on account of alleged coverage of the otherportion within the Nueva Vizcaya-Quirino Civil Reservation under Proc. 1498 of 11 September1975.

On 15 July 1991 Executive Director Leonardo A. Paat rejected petitioner‘s ProspectingPermit Application (PPA) on the ground that the Memorandum of July 08, 1991 endorsed by theRegional Technical Director for Mines revealed that the area covered was outside governmentreservation; that the prospect claim was in conflict with existing claims; and, that the area hadbeen extensively explored in the early 1980's.

Petitioner moved for reconsideration. Regional Executive Director Samuel Paragasrecommended to the DENR Secretary that petitioner's request for reconsideration be denied;that the existing rights of mining claim holders be respected; and, that the prior legal rightsof MPSA/Financial and Technical Assistance Agreement applicants over subject area berecognized.

 As regards petitioner's PPA filed with the DAR, it appeared that it was issued aclearance to prospect for six (6) months from December 11, 1995.

On August 15, 1997 petitioner appealed to public respondent Mines Adjudication Board(MAB). Petitioner maintained that subject area was within the Magat River Forest Reservation.

On June 11, 1998 the rejection of the PPA was affirmed whereas the mining claims ofrespondents Alberto G. Bumolo et al. that had been converted into a MPSA, subject tocompliance with R.A. 7942 and DAO No. 96-40, were given due course.

Petitioner moved for reconsideration. Respondent MAB denied petitioner‘s motion . 

Page 48: Envi Case Digests 3.1

7/24/2019 Envi Case Digests 3.1

http://slidepdf.com/reader/full/envi-case-digests-31 48/51

ISSUE:

Whether respondent MAB erred in finding that the area subject of the PPA was outsidethe Magat River Forest Reservation.

HELD:

Respondent MAB correctly upheld the ratiocination of Regional Executive DirectorParagas in denying petitioner's PPA.

The disapproval of Marcopper‘s PPA moreover, did not emanate from a singlerecommendation of the RTD for Mines. Records would show that as early as May 31, 1989 x xx the Bumolo group of PD 463 claims which Marcopper has eventually surrounded by filing itsown PAO 1-30 group of claims x x x x was confirmed by the Forest Engineering Section of theregion to be outside proclaimed watershed areas, wilderness, national parks and existinggovernment reforestation projects x x x x

In other words, the circumstance that the area covered by petitioner's PPA is outside the

Magat River Forest Reservation has been adequately established by the following evidence: (a)confirmation as early as 31 May 1989 by the Forest Engineering Section of Tuguegarao,Cagayan; (b) the 8 July 1991 Memorandum Report of Regional Technical Director Punsal Jr.;and, (c) plotting provided by the National Mapping and Resources Information Authority per its 2June 1995 indorsement of the maps to the office of the Regional Executive Director. Petitionercontests the exclusion of the area subject of its PPA within the Magat River Forest Reservationbased merely on the alleged "typographical error committed by somebody in the EngineeringSection of the DENR." Aside from the fact that the allegation does not have anything to supportit, the aforementioned documents which the Regional Executive Directors relied upon in denyingthe PPA had already settled the issue.

Furthermore, respondent MAB even fortified the bases for the rejection of petitioner's

PPA. As plotted by the Lands Management Sector of DENR Region 2 contained in the sketchplan of 11 November 1996 and as shown in the Land Use map of the Community Environmentand Natural Resources Office of Dupax, Nueva Vizcaya, the area covered under the PPA isindeed outside any government reservation.

Page 49: Envi Case Digests 3.1

7/24/2019 Envi Case Digests 3.1

http://slidepdf.com/reader/full/envi-case-digests-31 49/51

25.

G.R. No. 195580 April 21, 2014NARRA NICKEL MINING AND DEVELOPMENT CORP., TESORO MINING AND

DEVELOPMENT, INC., and MCARTHUR MINING, INC., Petitioners, vs. REDMONT

CONSOLIDATED MINES CORP., Respondent.

FACTS:

Sometime in December 2006, respondent Redmont Consolidated Mines Corp. (Redmont), adomestic corporation organized and existing under Philippine laws, took interest in mining andexploring certain areas of the province of Palawan. After inquiring with the Department ofEnvironment and Natural Resources (DENR), it learned that the areas where it wanted toundertake exploration and mining activities where already covered by Mineral ProductionSharing Agreement (MPSA) applications of petitioners Narra, Tesoro and McArthur.

Petitioner McArthur Narra and Tesoro, filed an application for an MPSA and Exploration Permit(EP) which was subsequently issued.

On January 2, 2007, Redmont filed before the Panel of Arbitrators (POA) of the DENR three (3)separate petitions for the denial of petitioners‘ applications for MPSA.

Redmont alleged that at least 60% of the capital stock of McArthur, Tesoro and Narra areowned and controlled by MBMI Resources, Inc. (MBMI), a 100% Canadian corporation.Redmont reasoned that since MBMI is a considerable stockholder of petitioners, it was thedriving force behind petitioners‘ filing of the MPSAs over the areas covered by applicationssince it knows that it can only participate in mining activities through corporations which aredeemed Filipino citizens. Redmont argued that given that petitioners‘ capital stocks were mostlyowned by MBMI, they were likewise disqualified from engaging in mining activities throughMPSAs, which are reserved only for Filipino citizens.

Petitioners averred that they were qualified persons under Section 3(aq) of Republic Act No.(RA) 7942 or the Philippine Mining Act of 1995. They stated that their nationality as applicants isimmaterial because they also applied for Financial or Technical Assistance Agreements (FTAA)denominated as AFTA-IVB-09 for McArthur, AFTA-IVB-08 for Tesoro and AFTA-IVB-07 forNarra, which are granted to foreign-owned corporations. Nevertheless, they claimed that theissue on nationality should not be raised since McArthur, Tesoro and Narra are in fact PhilippineNationals as 60% of their capital is owned by citizens of the Philippines.

On December 14, 2007, the POA issued a Resolution disqualifying petitioners from gainingMPSAs. The POA considered petitioners as foreign corporations being "effectively controlled"by MBMI, a 100% Canadian company and declared their MPSAs null and void.

Pending the resolution of the appeal filed by petitioners with the MAB, Redmont filed aComplaint with the Securities and Exchange Commission (SEC), seeking the revocation of thecertificates for registration of petitioners on the ground that they are foreign-owned or controlledcorporations engaged in mining in violation of Philippine laws.

CA found that there was doubt as to the nationality of petitioners when it realized that petitionershad a common major investor, MBMI, a corporation composed of 100% Canadians. Pursuant tothe first sentence of paragraph 7 of Department of Justice (DOJ) Opinion No. 020, Series of

Page 50: Envi Case Digests 3.1

7/24/2019 Envi Case Digests 3.1

http://slidepdf.com/reader/full/envi-case-digests-31 50/51

2005, adopting the 1967 SEC Rules which implemented the requirement of the Constitution andother laws pertaining to the exploitation of natural resources, the CA used the "grandfather rule"to determine the nationality of petitioners.

In determining the nationality of petitioners, the CA looked into their corporate structures andtheir corresponding common shareholders. Using the grandfather rule, the CA discovered that

MBMI in effect owned majority of the common stocks of the petitioners as well as at least 60%equity interest of other majority shareholders of petitioners through joint venture agreements.The CA found that through a "web of corporate layering, it is clear that one common controllinginvestor in all mining corporations involved x x x is MBMI." Thus, it concluded that petitionersMcArthur, Tesoro and Narra are also in partnership with, or privies-in-interest of, MBMI.

ISSUE:

Whether or not the Court of Appeals‘ ruling that Narra, Tesoro and McArthur are foreign  corporations based on the "Grandfather Rule" is contrary to law, particularly the expressmandate of the Foreign Investments Act of 1991, as amended, and the FIA Rules.

HELD:

No. There are two acknowledged tests in determining the nationality of a corporation: the controltest and the grandfather rule. Paragraph 7 of DOJ Opinion No. 020, Series of 2005, adoptingthe 1967 SEC Rules which implemented the requirement of the Constitution and other lawspertaining to the controlling interests in enterprises engaged in the exploitation of naturalresources owned by Filipino citizens, provides:

Shares belonging to corporations or partnerships at least 60% of the capital of which is ownedby Filipino citizens shall be considered as of Philippine nationality (CONTROL TEST), but if thepercentage of Filipino ownership in the corporation or partnership is less than 60%, only thenumber of shares corresponding to such percentage shall be counted as of Philippine

nationality (GRANDFATHER RULE). Thus, if 100,000 shares are registered in the name of acorporation or partnership at least 60% of the capital stock or capital, respectively, of whichbelong to Filipino citizens, all of the shares shall be recorded as owned by Filipinos. But if lessthan 60%, or say, 50% of the capital stock or capital of the corporation or partnership,respectively, belongs to Filipino citizens, only 50,000 shares shall be counted as owned byFilipinos and the other 50,000 shall be recorded as belonging to aliens.

The grandfather rule, petitioners reasoned, has no leg to stand on in the instant case since thedefinition of a "Philippine National" under Sec. 3 of the FIA does not provide for it. They furtherclaim that the grandfather rule "has been abandoned and is no longer the applicable rule." Theyalso opined that the last portion of Sec. 3 of the FIA admits the application of a "corporatelayering" scheme of corporations. Petitioners claim that the clear and unambiguous wordings of

the statute preclude the court from construing it and prevent the court‘s use of discretion inapplying the law. They said that the plain, literal meaning of the statute meant the application ofthe control test is obligatory.

SC disagreed. "Corporate layering" is admittedly allowed by the FIA; but if it is used tocircumvent the Constitution and pertinent laws, then it becomes illegal. Further, thepronouncement of petitioners that the grandfather rule has already been abandoned must bediscredited for lack of basis.

Page 51: Envi Case Digests 3.1

7/24/2019 Envi Case Digests 3.1

http://slidepdf.com/reader/full/envi-case-digests-31 51/51

Petitioners McArthur, Tesoro and Narra are not Filipino since MBMI, a 100% Canadiancorporation, owns 60% or more of their equity interests. Such conclusion is derived fromgrandfathering petitioners‘ corporate owners, namely: MMI, SMMI and PLMDC. The "controltest" is still the prevailing mode of determining whether or not a corporation is a Filipinocorporation, within the ambit of Sec. 2, Art. II of the 1987 Constitution, entitled to undertake theexploration, development and utilization of the natural resources of the Philippines. When in the

mind of the Court there is doubt, based on the attendant facts and circumstances of the case, inthe 60-40 Filipino-equity ownership in the corporation, then it may apply the "grandfather rule."